[Ответить в тред] Ответить в тред

02/12/16 - Конкурс визуальных новелл доски /ruvn/
15/11/16 - **НОВЫЙ ФУНКЦИОНАЛ** - Стикеры
09/10/16 - Открыта доска /int/ - International, давайте расскажем о ней!

Check this out!


Новые доски: /2d/ - Аниме/Беседка • /wwe/ - WorldWide Wrestling Universe • /ch/ - Чатики и конфочки • /int/ - International • /ruvn/ - Российские визуальные новеллы • /math/ - Математика • Создай свою

[Назад][Обновить тред][Вниз][Каталог] [ Автообновление ] 586 | 51 | 167
Назад Вниз Каталог Обновить

Тред тупых вопросов #52 Аноним 05/06/16 Вск 17:52:40  298225  
(61Кб, 600x781)
(95Кб, 533x347)
(29Кб, 604x361)
(307Кб, 1920x1200)
Тред вопросов о жизни, Вселенной и всем таком.

Спрашиваем то, за что в других местах выдают путёвку в биореактор. Здесь анонимные ученые мирового уровня критически рассмотрят любые гениальные идеи и нарисованные в Paint схемы.


Прошлый тред https://2ch.hk/spc/res/295207.html
Аноним 05/06/16 Вск 18:13:03  298229
>>298225 (OP)
Ура, с перекотом! ОП, ты молодец. У тебя нет пака с котами, которых ты делаешь для каждого перекота?
Аноним 05/06/16 Вск 18:19:15  298230
>>298229
Нет. Коты подряд случайно получились. Это не более чем совпадение.
Аноним 05/06/16 Вск 19:37:58  298234
(50Кб, 604x413)
>>298225 (OP)
> Спрашиваем то, за что в других местах выдают путёвку в биореактор.



Суть вопроса: образование Солнечной системы. Мы имеем изначально коллапс молекулярного облака, уже содержащего тяжелые элементы. Откуда они? Уран тот же самый, торий с триллионами лет периода полураспада. Как следствие вспышки древних звезд. Но образовать уран и торий - это надо ебануть весьма неслабо, т.е. вспышка была сверхновой. Это я нагуглил в вики. Внимание вопрос: где остаток сверхновой? Либо Черная дыра, либо Нейтронная звезда обязаны быть где-то рядом.
Аноним 05/06/16 Вск 19:42:12  298236
>>298234
> Либо Черная дыра, либо Нейтронная звезда обязаны быть где-то рядом.
Это минимум лет 20 (галактических) назад было, там давно перемешалось все уже.
Аноним 05/06/16 Вск 19:48:01  298238
>>298236
Что перемешалось? Вспыхнула сверхновая, образовалась ЧД в центре туманности. Из выброшенных химэлементов формируется солнечная система. Но труп предыдущей звезды неуничтожим в принципе в силу своей массы и деваться ему некуда же. И гравитация у него нехуевая, чтобы не оказывать влияния на формирования дочерних объектов.
Аноним 05/06/16 Вск 20:03:13  298240
(75Кб, 604x591)
>>298234
>Откуда они?
Из взрывов сверхновых, конечно.
Наше солнце из населения I, то есть к моменту образования уже сменилось население III и II.
Во-вторых звезды редко образовываются по одиночке, локальное место активного звездообразования называется звездными кластерами.
>обязаны быть где-то рядом.
Нет, не обязаны.
Звездные "ясли" нашего солнца не находились там, где сейчас находится Солнце. Звезды мигрируют по Галактике. Есть различные модели описания этих миграций. Возможно "сестры" нашего солнца размазаны по всему диску. Поиском родственных звезд по схожему химическому составу сейчас занимается GAIA.
http://www.cosmos.esa.int/web/gaia/iow_20160204
Аноним 05/06/16 Вск 20:07:04  298242
>>298238
> Что перемешалось?
Дом Облонских Окрестности Солнца. Если ты посмотришь на то, каким небо было 50к лет назад (1/4 года), ты найдешь очень много изменений. Все потому, что соседние звезды слабо связаны гравитационно и двигаются вокруг центра галактики неодинаково. А теперь экстраполируй это на промежуток в 80 раз (самый минимум, а может и в 160 раз) больший. За это время та нейтронная звезда могла за полгалактики улететь. И это если она была одна, что, скорее всего, неверно (хотя бы потому, что сомнительно, чтобы вся СС могла образоваться из остатков только одной звезды, вещества не хватит). СС образовалась из остатков кучи звезд II и III поколений, каких именно - сомневаюсь, что это когда-то определят.
Аноним 05/06/16 Вск 20:15:04  298243
(27Кб, 500x223)
>>298240
>"сестры" нашего солнца
Oh gosh, how cute & adorable :3 Наша Звезда не одинока и у неё есть родные родственники! Надеюсь Гая найдёт их наконец и мы сможем передать Солнцу их астрофоточки!
Аноним 05/06/16 Вск 20:17:25  298244
>>298242
> вся СС могла образоваться из остатков только одной звезды, вещества не хватит
Масса всего вещества СС = 0.001 от массы Солнца, лол. А если ебанул голубой гигант какойнть в 10 солнечных масс, то хватит и не на одну СС.

> За это время та нейтронная звезда могла за полгалактики улететь
А окружающая ее планетарная туманность на месте что ли осталась? Мы же наблюдаем древние остатки новых, там идет звездообразование и ЧД/НЗ в центре никуда не девается. Мне кажется ответ таков, что размер туманности-трупа в поперечнике 1000 св.лет. И в дальней области ее из сгустка мусора формировалась СС. Таким образом, ЧД никуда не делась и поди висит гденть в ебенях. Я просто не учел размера туманности: "туманность имеет диаметр в 11 световых лет (3,4 пк) и расширяется со скоростью около 1500 километров в секунду"


Аноним 05/06/16 Вск 20:22:18  298245
>>298243
> Надеюсь Гая найдёт их наконец и мы сможем передать Солнцу их астрофоточки!
И потом мы будем все вместе петь и танцевать!
https://www.youtube.com/watch?v=d0_zis_LwUY
Аноним 05/06/16 Вск 20:25:53  298246
>>298244
>Мы же наблюдаем древние остатки новых
Во-первых не новых, а сверхновых. Это нихуя не одно и тоже.
>там идет звездообразование
Во-вторых в остатках сверхновых звездообразование не идет, но ударная волна от сверхновой может вызывать уплотнение в ближайшем газопылевом облаке, создавая неоднородности и уплотнения, и запуская таким образом процесс звездообразования.
Аноним 05/06/16 Вск 20:27:27  298247
>>298244
> Масса всего вещества СС = 0.001 от массы Солнца, лол. А если ебанул голубой гигант какойнть в 10 солнечных масс, то хватит и не на одну СС.
Ты учти, что даже если звезда потеряет вообще всю свою массу (но тогда и НЗ не будет, и ЧД, и вообще нихуя), она не останется в пространстве небольшим сгустком, а полетит непрерывно расширяющейся сферой, которой самой по себе просто не хватит, чтобы схлопнуться в что-то с массой Солнца. А вот когда ударная волна от такого пройдет через газопылевое облако, она может и спровоцировать в нем нарастание неоднородностей и всплеск звездообразования, заодно и металлами обогатит.
Кстати, не совсем тупой вопрос: какую часть массы сверхновой составляют сбрасываемые оболочки? Я вроде когда-то читал, что достаточно небольшую, но сходу пруфов не нашел.
> Таким образом, ЧД никуда не делась и поди висит гденть в ебенях.
Только эти ебени уехали от нас черт знает куда, возможно - вообще в другой рукав или в другую половину диска Галактики.
Аноним 05/06/16 Вск 20:57:47  298251
>>298247
>непрерывно расширяющейся сферой
Скорее эти остатки напоминают раковые опухоли, всех размеров и форм, но в общем ты все правильно написал.
>которой самой по себе просто не хватит, чтобы схлопнуться в что-то с массой Солнца
Оно в принципе не может схлопнуться, оно разлетается в разные стороны с огромной скоростью.
>какую часть массы сверхновой составляют сбрасываемые оболочки?
Очень поразному бывает. Зависит от типа сверхновой и исходной массы. Масса пульсара в Крабовидной туманности 1,4-1,6 М☉ при этом сама туманность оценивается в 4.6±1.8 M☉.


Аноним 05/06/16 Вск 21:08:44  298254
(420Кб, 1024x1024)
Все мы со школы знаем о реликтовом излучении, ныне имеющему температуру 3К. Вселенная остывала с расширением и значит прошла тот момент, когда ВСЯ была заполнена излучением 300К, т.е. с температурой теплого летнего утра. Может как раз тогда все было кругом заполнено жизнью, были изобретены прон и двачи и потом благополучно было растащено по атомам на миллиарды световых лет. И т.е. сейчас искать жизнь в дохлой ледяной расширяющейся Вселенной - поди тупо, когда весь экшон и звездные войны давно закончились и никому тут больше ничего не надо, поэтому кроме нас дураков никого тут нету больше?
Аноним 05/06/16 Вск 21:14:24  298255
>>298251
> Оно в принципе не может схлопнуться, оно разлетается в разные стороны с огромной скоростью.
Ну, я счел себя недостаточно компетентным в этом вопросе, чтобы отвергать возможность влияния гравитации на отдельные флуктуации плотности. Другое дело, что я не могу сходу оценить возможность того, что гравитация способна остановить их изначальный разлёт.
> Очень поразному бывает. Зависит от типа сверхновой и исходной массы. Масса пульсара в Крабовидной туманности 1,4-1,6 М☉ при этом сама туманность оценивается в 4.6±1.8 M☉.
Спасибо.
Аноним 05/06/16 Вск 21:50:46  298257
>>298254
>с температурой теплого летнего утра. Может как раз тогда все было кругом заполнено жизнью, были изобретены прон и двачи
И с очень приличной величиной постоянной Хаббла, а самое главное - пиздецовыми излучениями огромных мощностей, везде и всюду, включая всеразличные жесткие рентгены, не очень любимые сракой лысых макак.
Аноним 05/06/16 Вск 21:50:55  298258
>>298225 (OP)
Посоны, захотел обмазаться наблюдением звезд и всяческих объектов, посему реквестую програм, чтива и прочей инфы по этой теме.
Исходные данные - знаю Большую и Малую Медведицу и точно определяю Полярную звезду. С некоотрым сомнением Пояс Ориона, были сомнения насчета Венеры и Марса.
Аноним 05/06/16 Вск 22:02:05  298261
(31Кб, 482x480)
>>298245
кекеке
Аноним 05/06/16 Вск 22:05:27  298262
>>298258
Бля, пора бы faq для таких нюфань завести. Слишком часто спрашивают.
Аноним 05/06/16 Вск 22:05:42  298263
>>298234
Из Солнца. Планеты вынырнули из Солнца, спутники вынырнули из планет.
Аноним 05/06/16 Вск 22:08:32  298264
>>298254
Вселенная не расширяется. Гугли утомленный свет.
Аноним 05/06/16 Вск 22:10:20  298265
>>298263
Лолвут. Планеты никогда не были частью Солнца. И Солнце и планеты образовались из одного газопылевого облака - то есть они ровесники. А вот со спутниками разная история и разные теории существуют на этот счёт. По крайней мере известно точно, что спутники могут как захватываться в готовом виде чужой планетой, так и могут быть "рождены" из общей с планетой массы вещества, через столкновение с другим небесным телом.
Аноним 05/06/16 Вск 22:11:43  298266
>>298264
Кефирщиков выгнали, теперь ты вместо них?
Аноним 05/06/16 Вск 22:31:04  298269
>>298265
Насколько мне известно небулярную гипотезу образования планет сегодня никто серьезно не рассматривает. Не знаю, почему она тут в почете. Наверное учебники еще не исправили.
Аноним 05/06/16 Вск 23:07:31  298274
>>298269
>никто серьёзно не рассматривает
Ммм правда? А у меня противоположные данные.
>Наверное учебники еще не исправили.
Ну наверное. А ты какой гипотезы придерживаешься?
Аноним 06/06/16 Пнд 00:56:30  298281
Как мы знаем, анонисмус обычно всегда не согласен с чем-либо. Ну вот. Опровергните эту безысходность.
https://www.youtube.com/watch?v=ZL4yYHdDSWs
Аноним 06/06/16 Пнд 01:38:59  298285
Поясните, пожалуйста, обстоятельно за Луну. Можно ссылкой на книгу или видео, можно словами. Всю жизнь я воспринимал её наличие на небе как данность, а тут вдруг внезапно осознал, что она на самом деле относительно редко видна на небе, движется как-то неравномерно и вообще я совершенно не понимаю закономерности её хождения по небу. После чтения википедии и долгого наблюдения за Луной в стеллариум осталось чувство замешательства и желание увидеть внятный чертёж с описанием движения Луны вокруг Земли.

Итак, чего я не понимаю. Есть 3 циклические величины: высота Луны над горизонтом в данной точке, расстояние от Луны до Земли и фаза Луны. Плюс, бонусный пункт, приливный цикл. Я не понимаю связи между ними. Например, сейчас я смотрю Луна ночью почти всё время под Землёй. Это зависит лишь от фазы Луны, или Луна ходит по разным маршрутам на небе в зависимости ещё от какого-то цикла? Расстояние между центрами тел меняется с тем же циклом, что и фаза луны, или нет? Прилив это целиком суточное явление или влияние тяготения Луны в данном месте меняется ещё от чего-то, скажем, всё с той же фазой?
Аноним 06/06/16 Пнд 02:17:21  298288
>>298258
Учебник по астрономии + Стеллариум
Аноним 06/06/16 Пнд 02:19:27  298289
>>298265
Не ровесники, хоть и из одного облака и даже локальной его части. Образование планет произошло после рождения солнца.
Аноним 06/06/16 Пнд 04:34:48  298292
>>298281
Где ты в этом видео безысходность разглядел, долбоебина? Все кончается тем, что типа все галактики улетят от млечного пути и будем мы сычевать в одной галактике, но у нас еще типа будет триллион звезд для освоения и покорения. А суть теории большого разрыва в том, что на разлете галактик все не остановится. Рано или поздно постоянная Хаббла достигнет запредельных значений и сначала структура галактик будет нарушена, потом структура звезд, затем планет, затем нейтронных звезд, затем человеков. Последовательность не точная, но суть в том, что через многие миллиарды лет сама природа вселенной разорвет всю материю на атомы, а потом атомы на нуклоны, нуклоны на кварки, кварки на преоны, преоны на хуйоны, хуйоны на пиздены и все, конец, одни только черные дыры останутся темными островками хуй пойми чего, потому что природа их также недостаточно изучена, и не ясно, может ли их разорвать на куски сверхбыстрое расширение. Пока считается, что не сможет, но тут такое.
Но это только одна сторона медали. Есть одна омега, параметр космологического состояния или как-то так называется. Ее значение и должно определить дальнейшую судьбу мироздания текущего вида. И значение оной может оказаться недостаточным для большого разрыва. Но в таком случае вселенную ждет коллапс в одну большую сингулярность. Для человеков и их атомов сие хорошего также не сулит. И даже если сущность вселенной в том, что она в вечном "сердцебиении" будет поддерживать какое-то промежуточное состояние, не позволяющее барионной материи сколлапсировать или разъебениться на самые крошечные составляющие, то принципы термодинамики никто не отменял. Эти принципы сулят псевдостабильной вселенной через очень очень много времени т. н. тепловую смерть. Когда вся материя придет к тепловому балансу. То есть среднее значение температуры примет вся материя во вселенной. Температура эта не курортная. Совсем. Даже жидкий азот на холод жаловаться будет.
>Как мы знаем, анонисмус обычно всегда не согласен с чем-либо
Хуйню спизданул. Тут научным подходом упарываются ващет.
Аноним 06/06/16 Пнд 08:18:02  298296
>>298225 (OP)
Если на газовый гигант завести баллон с кислородом, масляную тряпку и воспламенить - получится? Почему нет? Какие подводные камни?
Аноним 06/06/16 Пнд 08:24:14  298297
(829Кб, 1916x1076)
>>298296
Аноним 06/06/16 Пнд 10:17:03  298304
>>298292
>Рано или поздно постоянная Хаббла достигнет запредельных значений
>одни только черные дыры останутся темными островками хуй пойми чего
При достаточно большом значении постоянной Хаббла разорвет и их.
Аноним 06/06/16 Пнд 10:20:03  298305
>>298304
Бляяя, надо же что-то с этим делать.
Аноним 06/06/16 Пнд 10:23:13  298307
>>298305
Ну, может она вдруг перестанет расти. Такое уже бывало.
Аноним 06/06/16 Пнд 10:23:56  298308
>>298307
Надо изучать, как-то сделать чтобы перестала расти.
Аноним 06/06/16 Пнд 11:48:02  298316
(3840Кб, 636x360, 00:00:38)
Этих животных уже видели?
Аноним 06/06/16 Пнд 12:08:26  298318
>>298316
Да и хули? Долбоебов везде полно, а астрономию в школы все равно надо вернуть, конечно.
Аноним 06/06/16 Пнд 12:23:22  298319
(829Кб, 1916x1076)
>>298318
А её сейчас нет в школах?
Аноним 06/06/16 Пнд 12:33:20  298320
>>298319
Давно уже нет. Есть в курсе физики несколько учебных часов по верхам, вроде. В некоторых школах как факультатив или дополнительный предмет по желанию, но далеко не везде.
Аноним 06/06/16 Пнд 12:35:34  298321
>>298318
были потуги в прошлом году. НЕТ не хочу блядь, буду основы копрославия вводить
Кстати а чем аргументировали отмену изучения астрономии в 90-х?
Аноним 06/06/16 Пнд 12:38:14  298322
>>298321
Нинужно. Бедные детки и так перегружены а тут ещё какая-то астрономия никому не нужная. Есть же физика, туда 3 часа запихаем и хватит. Все равно учителей нет, потому что все из школ бегут, так как жрать нечего.
Аноним 06/06/16 Пнд 13:12:33  298326
(319Кб, 1916x1076)
(829Кб, 1916x1076)
>>298320
Аноним 06/06/16 Пнд 13:14:28  298327
>>298326
Да кому нахуй твоя сраная астрономия нужна? Полутора ботанам со всех параллелей? Кому интересно сам изучит что захочет, лучше даже получится чем у говноучителей.
Аноним 06/06/16 Пнд 13:15:28  298328
>>298318
Честно говоря, астрономия в школе была уныла, я её не любил, зафанател гораздо позже, после института.
Аноним 06/06/16 Пнд 13:32:38  298330
>>298296
Бамп вопросу
Аноним 06/06/16 Пнд 13:35:11  298331
>>298327
>>298328
От препода сильно зависит, как и с другими предметами. Но заинтересовать личинок можно было бы больше. Плюс в головах большего количества личинусов отложилось бы хоть немного, и они не несли бы хуету на камеру.
Аноним 06/06/16 Пнд 13:39:27  298333
>>298296
Лол, получится, конечно. Будет гореть, пока не выйдет кислород, а потом всё. Но если окружающая атмосфера гиганта будет состоять из чего-то типа водорода или метана, то у тебя не получится нормально поджечь тряпку - можно подорваться немного. Но большого взрыва, конечно, не будет. Будет ровно столько, на сколько хватило выпущенного кислорода.
Аноним 06/06/16 Пнд 14:34:14  298336
Извинити за тупость. Как вся эта движуха
https://www.youtube.com/watch?v=No0omeHIxwo
согласуется со стандартной моделью расширяющейся Вселенной? Особенно в том смысле, что часть галактик летит к нам, а не от нас. А равно и в том, что летят они по каким-то ебанутым кривым. В статье авторов внятного объяснения не нашёл (она вообще не об этом; но возможно, я просто жопочтец).
Аноним 06/06/16 Пнд 14:56:11  298339
>>298336
>Извинити за тупость. Как вся эта движуха
>https://www.youtube.com/watch?v=No0omeHIxwo
>летят они по каким-то ебанутым кривым. В статье авторов внятного объяснения не нашёл.
Жопой смотрел потому что. Там на графике отложены не координаты местоположения, а скорость блядь.
Аноним 06/06/16 Пнд 14:59:31  298340
>>298289
Ну там разница максимум 100 млн лет. Это ровесники в масштабе времени жизни солнечной системы.
Аноним 06/06/16 Пнд 15:05:19  298341
>>298285
Я хз, что там может быть непонятно после чтения википедии. Даю ссылку на английскую статью, потому что в русской версии всего один абзац
https://en.wikipedia.org/wiki/Orbit_of_the_Moon
Алсо вот здесь объясняются фазы Луны
http://www.moonconnection.com/moon_phases.phtml
И вообще советую полуркать по этому сайту - он целиком посвящён Луняшке
Аноним 06/06/16 Пнд 16:13:17  298348
Почему многие статьи по астрономии в вики на русском - откровенное гавно с небольшим количеством текста и без картинок? Что если я просто засяду и буду переводить такую ж статью с английского на русский, с такими же картинками, почему до этого еще никто не догадался?
Аноним 06/06/16 Пнд 17:21:33  298352
(539Кб, 2560x1600)
Доставьте книг о крупномасштабной структуре вселенной, на русском. Одной Википедии мне недостаточно. С меня графон.

https://ru.wikipedia.org/wiki/%D0%9A%D1%80%D1%83%D0%BF%D0%BD%D0%BE%D0%BC%D0%B0%D1%81%D1%88%D1%82%D0%B0%D0%B1%D0%BD%D0%B0%D1%8F_%D1%81%D1%82%D1%80%D1%83%D0%BA%D1%82%D1%83%D1%80%D0%B0_%D0%92%D1%81%D0%B5%D0%BB%D0%B5%D0%BD%D0%BD%D0%BE%D0%B9
Аноним 06/06/16 Пнд 17:24:43  298353
А черная дыра движется в пространстве или на месте стоит?
Аноним 06/06/16 Пнд 17:29:35  298354
>>298316
Я отказываюсь в такое верить.
Аноним 06/06/16 Пнд 17:34:38  298355
Какое было самое важное научное открытие связанное с космосом за последние 20 лет?
Аноним 06/06/16 Пнд 17:39:19  298356
>>298355
Гравитационные волны мейби.
Аноним 06/06/16 Пнд 17:46:44  298357
>>298352
>книг о крупномасштабной структуре вселенной, на русском
Таких нит. Что-то можно узнать из различных книг, посвящённых другим темам. Например, "Почему небо чёрное". Но чтобы прям вот целая книга про это - что-то не припомню.
Вот в этой статье
https://ru.wikipedia.org/wiki/%D0%9C%D0%B5%D1%82%D0%B0%D0%B3%D0%B0%D0%BB%D0%B0%D0%BA%D1%82%D0%B8%D0%BA%D0%B0
приводятся ссылки на чтиво в примечаниях. Глянь там.
Аноним 06/06/16 Пнд 17:47:47  298358
>>298353
Относительно чего?
Аноним 06/06/16 Пнд 17:53:29  298359
>>298358
Относительно солнца.
Аноним 06/06/16 Пнд 18:03:41  298360
>>298339
>1:20
>Gravitational peculiar motions towards us are in blue, and motions away are in red.
>towards us
>away
>скорость блядь
Скорость блядь, но вместе с направлением движения. Или я что-то опять не так понял?
Аноним 06/06/16 Пнд 18:27:21  298361
>>298355
Бозон Гиггза.
Аноним 06/06/16 Пнд 18:30:26  298362
>>298361
Почему же тогда об этом бозоне слышал только ты?
Аноним 06/06/16 Пнд 18:34:06  298363
>>298360
Да нет, это он не правильно понял. Синее смещение это именно сближение в самом что ни наесть прямом смысле. Только в чем ты противоречие с моделью расширяющейся вселенной видишь? Постоянная Хаббла что-то около 73 км/с на МПк. Та же Андромеда находится от нас в около 0,7ми МПк. То есть по модели расширения приблизительно 50 км в линейном протяжении пространства прирастает между галактиками ежесекундно. Но это не мешает им сближаться с большей скоростью из-за действия взаимных гравитационных сил. Такой же вариант работает и для больших групп, ибо пока значения постоянной Хаббла не критически велики.
Аноним 06/06/16 Пнд 18:36:07  298364
>>298362
Кокой ты у мамы формалист однако.
Аноним 06/06/16 Пнд 18:45:39  298365
>>298355
Расширение Вселенной с ускорением.
>>298356
Тоже неплохо. Но не совсем про космос, а про физику вообще.
>>298361
Тоже не совсем профильное открытие.
Аноним 06/06/16 Пнд 18:46:18  298366
>>298357
Нашёл там пару годных статей. Спасибо.
Аноним 06/06/16 Пнд 18:47:41  298367
>>298348
Так много статей сделано, тупо перевод, или сокращенный вариант. Просто количество людей, которые этим занимаются очень немного.
Аноним 06/06/16 Пнд 18:48:15  298368
>>298353
Во Вселенной все движется.
Аноним 06/06/16 Пнд 18:51:46  298369
>>298360
>peculiar motions
Погугли сначала.
>1:20
Там нет ебанутых траекторий.
Аноним 06/06/16 Пнд 19:17:27  298371
Что такое вакуум?
Аноним 06/06/16 Пнд 19:23:56  298372
>>298316
Всё нормально ведь.
Альфа Кентавра - самая близкая видная невооруженным взглядом.
Марс - самая близкая "блуждающая звезда".
Солнце тоже звезда.
Аноним 06/06/16 Пнд 19:39:00  298373
>>298371
Пространство свободное от вещества. То есть вакуум не обязательно пуст, а может содержать всякую невещественную материю не стоит вскрывать и даже вещество в низкой концентрации.
Аноним 06/06/16 Пнд 19:41:26  298374
>>298373
Что это и как это?
Аноним 06/06/16 Пнд 19:41:49  298375
>>298374
>может содержать всякую невещественную материю
Что это и как это?
Аноним 06/06/16 Пнд 19:55:41  298377
Как происходит отсоединение ступеней ракеты, куда они потом деваются?
Аноним 06/06/16 Пнд 19:58:40  298378
Есть ли альтернативный способ доставки людей в космос?
Без использования ракет.
Аноним 06/06/16 Пнд 20:06:52  298379
Что будет с человеком если поместить его в вакуум но дать ему доступ к кислороду через балон(или другим любым способом, короче чтобы дышал)?
Аноним 06/06/16 Пнд 20:11:10  298380
(363Кб, 887x320)
>>298377
>куда они потом деваются?
А сам как думаешь? Падают, ёпт. Недогоревшие обломки потом оказываются где-нибудь в тайге, в глухой степи или посреди океана.
Аноним 06/06/16 Пнд 20:17:32  298381
>>298380
Они могут гореть до самого приземления или нет?
Аноним 06/06/16 Пнд 20:19:16  298383
>>298375
Это https://2ch.hk/spc/res/223626.html
Аноним 06/06/16 Пнд 20:21:19  298384
>>298379
Лягушку надувал когда нибудь?
Аноним 06/06/16 Пнд 20:21:59  298385
(14Кб, 378x226)
>>298377
>Как происходит отсоединение ступеней ракеты
Механически. Защелки стянуты пироболтами, те срабатывают, защелки открываются, срабатывают толкатели (если есть). На некоторых ракетах ставят еще мелкие твердотопливные движки, чтобы увести ракету от ступени. Иногда делают горячее разделение (следующая ступень включается раньше разделения), чтобы ракету не порвало резким обнулением нагрузки. Иногда холодное.

>куда они потом деваются?
Нижние - падают твоей мамке на голову. Верхние, орбитальные - иногда сводятся назад обратным импульсом, но чаще всего так и остаются болтаться наверху, т.к. горючки вечно в обрез. Некоторые ступени от лунных Сатурнов до сих пор курсируют между Луной и Землей. Некоторые, выводившие АМС, болтаются на солнечной орбите. Обычно верхние ступени при выводе ИСЗ уводят на орбиту захоронения и пассивируют (стравливают всё что там есть под давлением), если есть возможность.
Аноним 06/06/16 Пнд 20:24:00  298386
>>298384
Бля, нитраль, ща начнётся опять.
Вообще, почему ответ на этот вопрос до сих пор не в шапке?
Аноним 06/06/16 Пнд 20:24:44  298387
>>298384
>>298386
Нет. Так что будет? Откуда инфа?
Аноним 06/06/16 Пнд 20:26:26  298388
(52Кб, 1600x1067)
(88Кб, 640x356)
>>298381
Могут и горят. Фейерверк зависит от скорости и высоты, которую успела набрать ступень. Вот например от промежуточных ступеней Союзов получается такой.
Аноним 06/06/16 Пнд 20:27:43  298389
>>298388
Бля, K P A C U B O. А потом это с долей вероятности устраивает пожары?
Аноним 06/06/16 Пнд 20:28:21  298390
>>298379
Разрыв легких (как минимум) и пиздарики. Запомни, двачер: чтобы выжить в вакууме, выдохни!
Аноним 06/06/16 Пнд 20:29:18  298391
>>298390
Но это ведь теория, так? Ни на ком не проверено?
Аноним 06/06/16 Пнд 20:29:57  298392
>>298389
Типы из поисковых групп говорят, что пожары от них при падении в лес это обычное дело.
Аноним 06/06/16 Пнд 20:31:39  298393
(94Кб, 511x350)
>>298391
Проверено, но лишь один раз. Впрочем, ты всегда можешь помочь науке, проведя подтверждающий эксперимент на натуре с собственным участием.
Аноним 06/06/16 Пнд 20:32:26  298394
>>298391
Думаю на свиньях могли проверить
Аноним 06/06/16 Пнд 20:33:19  298395
(22Кб, 482x357)
https://www.youtube.com/watch?v=ClVwfryyyJo
Блять, это мы так летим. Куда мы летим то? Или вся наша галактика тоже двигается по спирали, а скопление галактик тоже? И все они куда-то ебашут?
Аноним 06/06/16 Пнд 20:34:17  298396
>>298393
Это воссозданные условия космического вакуума? Они чем-то отличаются?
Аноним 06/06/16 Пнд 20:35:09  298398
Есть ли в космосе ветер? Может ли он сдуть нашу систему к куям?
Аноним 06/06/16 Пнд 20:38:07  298399
Что удерживает газы на планете?
Аноним 06/06/16 Пнд 20:39:22  298400
>>298396
Вакуумная камера для проверки скафандров.
>космического вакуума
Космический вакуум бывает разный, но даже чтобы откачать воздух до состояния низкой орбиты, нужны пиздецки специализированные насосы, т.к. в таком глубоком вакууме молекулы уже слишком редки, чтобы откачивать их обычными способами. Обычно такой откачкой не заморачиваются, достаточно не очень глубокой. Исключение - камеры для проверки каких-либо свойств аппаратуры, которая зависит от степени вакуумирования.

Уж тем более радиацию, ультрафиолет, поток ионов и электронов и т.п. смоделить нельзя никак, только пускать на орбиту.
Аноним 06/06/16 Пнд 20:39:45  298401
>>298399
Гравитация.
Аноним 06/06/16 Пнд 20:40:38  298402
Как попасть в космонавты?
Аноним 06/06/16 Пнд 20:41:32  298403
>>298398
Да, есть ветер от Солнца и он сдувает чуть по чуть атмосферу. Еще есть немного другой ветер. Есть мнение, что в древности он очень не хило раскручивал наше светило, благодаря чему от него отлетали куски, на одном из которых мы сейчас живем.
Аноним 06/06/16 Пнд 20:41:40  298404
>>298402
Нужно хорошо учиться, кушать, слушаться маму
Аноним 06/06/16 Пнд 20:44:41  298405
(70Кб, 1024x680)
>>298398
Есть много разных типов ветра, и даже погода есть. Но вообще это смотря что считать ветром и смотря что считать космосом.

В астрофизике/планетологии есть такое понятие как "экзотическая атмосфера", в него входят и молекулярные облака, и временные образования типа испаряющейся оболочки кометы (зачастую улетающей на больших скоростях от источника), и джеты, и т.п. Большинство экзотических атмосфер находятся в состоянии движения.
Аноним 06/06/16 Пнд 20:45:01  298406
>>298402
Нужно летное образование или большие деньги. Иди в курсанты военным летчиком.
Аноним 06/06/16 Пнд 20:48:00  298407
>>298406
В наши дни не нужно ни то, ни другое. Достаточно быть инженером, медиком или биологом, и естественно заниматься непосредственно связанными с космонавтикой проблемами. ОЧЕНЬ МНОГО заниматься. Плюс близкое к идеальному здоровье и отсутствие пломб в зубах.
Аноним 06/06/16 Пнд 20:49:40  298408
>>298407
>отсутствие пломб в зубах
Почему с пломбами нельзя?
Аноним 06/06/16 Пнд 20:50:02  298409
>>298407
Без зубов можно тоже
Аноним 06/06/16 Пнд 20:54:05  298410
Понятно.
Что из себя представляют облака?
Газ, получившийся при сгорании топлива от ракет куда потом девается в космосе? Куда он улетает?
Аноним 06/06/16 Пнд 20:58:02  298411
>>298410
Возможны ли облака в космосе?
Аноним 06/06/16 Пнд 21:00:36  298412
(7Кб, 271x186)
>>298411
Угу
Аноним 06/06/16 Пнд 21:01:08  298413
>>298408
В случае разгерметизации полости могут ёбнуть и ты можешь охуеть, а дезориентация в условиях аварийной ситуации никому не нужна. То же самое требование к лётчикам кстати, если не ошибаюсь. Существуют также специальные пломбы, не дающие полостей/газообразования, с ними можно.
Аноним 06/06/16 Пнд 21:04:27  298414
>>298412
Это что?
Аноним 06/06/16 Пнд 21:05:32  298415
>>298410
>Газ, получившийся при сгорании топлива от ракет куда потом девается в космосе? Куда он улетает?
Расширяется до выравнивания с окружающей средой.
Аноним 06/06/16 Пнд 21:06:28  298416
>>298415
Вакуумом?
Аноним 06/06/16 Пнд 21:07:16  298417
>>298414
облака, епт
Аноним 06/06/16 Пнд 21:07:32  298418
>>298414
Облака, белогривые лошадки водорода. В космосе. конкретно на пике - туманность Розетка, но такой хуйнеё галактики забиты доверху.
Аноним 06/06/16 Пнд 21:08:29  298419
>>298416
Полного вакуума нет нигде, даже в межзвездном пространстве, не говоря уже об околосолнечном или тем более поблизости любых планет.

(на самом деле в межзвездных облаках газ даже поплотнее будет, чем у нас на орбите)
Аноним 06/06/16 Пнд 21:21:11  298420
>>298367
т.е. я могу на чистом энтузиазме сидеть и перепиливать статьи? А меня не пидорнут и не откатят статью к первоначальному варианту?
Аноним 06/06/16 Пнд 21:25:09  298423
>>298420
>не пидорнут
Это как получится.
>не откатят статью к первоначальному варианту
Вряд ли, если английский у тебя не уровня "голый кондуктор бежит под вагоном".
Рувики - проект добровольный, как говорил Дядя Фред, и в нём приходится быть вежливым со всеми, включая иллюзионистов и клоунов, а не только с директором и утигрителями кротов.
Аноним 06/06/16 Пнд 21:25:17  298424
>>298420
Не исключено. Для правок видипедии вообще нужна холодная жопа. Достаточно посмотреть на статью "Терраформирование" или на математические статьи рукипедии, которые писали люди с формализмом головного мозга.
Аноним 06/06/16 Пнд 21:48:01  298429
>>298375
Темную материю например. Еще вакуум постоянно рождает т.н. виртуальные частицы.
Аноним 06/06/16 Пнд 21:52:08  298430
>>298395
Мы летим вокруг солнце, которое летит вокруг центра Галактики и еще перемещается вверх-вниз относительно плоскости галактического диска, Галактика летит навстречу галактики в Андромеде, все вместе мы летим в сторону Великого аттрактора. Вся эта ебала в сумме тоже движется хуй знает куда.
Аноним 06/06/16 Пнд 23:00:32  298440
>>298430
Галактики в нашем кластере вращаются вокруг общего центра масс, и эта конструкция тоже вокруг чего то вращается и летит. А вся эта хуйня ОЛОЛОПЛАНЕТЫ ВРАЩАЮТСЯ НЕ ПО КРУГУ по эллипсу, А ПА СПЕРАЛЕ пусть петушки из контача читают
Аноним 07/06/16 Втр 00:36:51  298460
>>298378
Космический лифт. Но это только концепция.
Аноним 07/06/16 Втр 00:42:49  298462
>>298375
Невещественная материя = поле.
Аноним 07/06/16 Втр 00:52:03  298465
(209Кб, 1500x1004)
Посоны, накидайте платиновых вопросов спейсача. Я запилю гайд как-нибудь. Пока вот что собрал:

ПЛАТИНА СПЕЙСАЧА:
- сколько планет в солнечной системе? а звёзд?
- что будет с голым человеком в открытом космосе? его разорвёт? он замёрзнет?
- с чего начать изучение астрономии?
- какой телескоп выбрать?
- как стать космонавтом?
Аноним 07/06/16 Втр 00:58:49  298466
>>298465
Лови:
> а давайте запилим шапку треду тупых вопросов
Аноним 07/06/16 Втр 01:03:12  298467
>>298466
двачую этого
Каждый тред приходит герой-одиночка, который грозится сделать фак, что бы все охуели.
Это само по себе платина. >>275090
Аноним 07/06/16 Втр 01:03:46  298468
>>298465
>какой телескоп выбрать?
Отдельный тред есть ващет.
Аноним 07/06/16 Втр 01:06:19  298469
>>298465
Про еблю и дрочку на орбите забыл
Аноним 07/06/16 Втр 01:17:12  298470
>>298468
Я это и хочу в ответах написать.
Аноним 07/06/16 Втр 01:18:09  298471
>>298467
>>298466
Да я как бы давно тут сижу и что-то не припомню, чтобы про шапку кто-то писал. Ну похуй. Не надо так не надо.
Аноним 07/06/16 Втр 01:24:10  298473
>>298471
Тебя в каждый такой пост в каждом треде еблом ткнуть надо, или ты сам справишься? Сидит он.
Аноним 07/06/16 Втр 01:28:59  298475
>>298473
Возможно пропустил что-то. Не только этот и далеко не каждый тупых-ответов-тред ведь читаю.
Аноним 07/06/16 Втр 01:45:05  298476
Что такое газовый гигант?
Аноним 07/06/16 Втр 01:47:12  298477
Откуда мы можем знать что теории о черных дырах, прочем говне, etc. верны?
Аноним 07/06/16 Втр 01:53:10  298478
>>298476
Юпитер и Сатурн. Планеты натурально состоящие из газов и от этого большие.
Аноним 07/06/16 Втр 01:57:59  298479
>>298477
Эти теории предсказали соответствующие объекты со всеми их свойствами. Наблюдения нашли объекты, которые идеально ложатся под предсказанные наблюдаемые признаки. А потом появилась куча альтернативных теорий, которые исходят из существования этой экзотики как данности.
Аноним 07/06/16 Втр 09:03:13  298500
>>298477
СТО позволяет это с высокой точностью предсказывать.

Аноним 07/06/16 Втр 09:06:52  298501
>>298467
>>298465
Вы бы их хоть на pastebin вынесли. Пока процедура принятия в Парижскую Палату Мер и Весов длится.
Аноним 07/06/16 Втр 09:41:23  298503
>>298412
>(7Кб, 271x186)
нахуй так шакалить, когда постишь туманности?
Аноним 07/06/16 Втр 09:42:22  298504
>>298465
как сделать телескоп бесплатно и без регистрации для наблюдения за Плутоном?
Аноним 07/06/16 Втр 09:42:59  298505
>>298500
СТО не идеальна и могут существовать различные исключения
Аноним 07/06/16 Втр 10:10:13  298507
>>298505
Очень многие физические теории не идеальны, и для них могут существовать различные исключения. Но пока лучшей теории не предложено.
Аноним 07/06/16 Втр 10:31:46  298508
Кстати, а что с аномалией "Пионера"? Всё же третья открытая проблема ОТО в солнечной системе. Русская википедия как всегда утверждает, что "мальчика не было", и приводит как истину в последней инстанции работу россиянских товарзчей, объясняющую всё тепловым излучением самих аппаратов. Англовики не так категорична, хотя тоже ставит эту теорию на первое место. Но дает несколько абзацев со спорными моментами, и говорит, что "срывающие покровы" статьи в периодике уже устарели, а новых так и нет (хотя проблема очень интересная).

Есть спецы на эту тему? Сами статьи:
https://ru.wikipedia.org/wiki/Эффект_«Пионера»
https://en.wikipedia.org/wiki/Pioneer_anomaly
Аноним 07/06/16 Втр 12:19:35  298521
>>298508
А что не так то? Статья с расчетами есть - бери да проверяй.
Аноним 07/06/16 Втр 12:21:07  298522
>>298505
Какие исключения? Например?
Аноним 07/06/16 Втр 19:13:30  298579
>>298521
>А что не так то? Статья с расчетами есть - бери да проверяй.
С тем же успехом ты мог предложить мне самостоятельно проверить доказательство теоремы о модулярности (гипотезы Шимуры-Таниямы).
Аноним 07/06/16 Втр 19:28:40  298582
Как сделать телескоп?
Аноним 07/06/16 Втр 19:40:42  298585
>>298582
Берешь слабо увеличивающую широкую лупу, берешь сильно увеличивающую мелкую лупу, смотришь в мелкую через крупную и ловишь расстояние на котором видна четкая увеличенная перевернутая картинка, делаешь по этому расстоянию тубус с регулятором длины, вставляешь в него линзы.
Аноним 07/06/16 Втр 19:41:54  298588
>>298585
Где брать линзы?
Аноним 07/06/16 Втр 19:46:23  298592
>>298588
В магазине канцтоваров, в бабушкиных очках, в старом фотоаппарате.
Аноним 07/06/16 Втр 19:47:29  298593
>>298585
>смотришь в мелкую через крупную
то есть наоборот, ну ты понел
Аноним 07/06/16 Втр 19:51:14  298595
>>298593
Не доверяю мелобуквенным.
Аноним 07/06/16 Втр 19:51:37  298596
>>298595
Мелкобуквенным*
Аноним 07/06/16 Втр 19:54:07  298598
>>298596
ТОГДА СПРОСИ НА ТЫТУБЕ
https://www.youtube.com/results?search_query=как+сделать+телескоп
Аноним 07/06/16 Втр 20:04:08  298600
>>298598
Отсыпь линз(
Аноним 07/06/16 Втр 21:08:40  298616
(147Кб, 1174x564)
Спейсаны, кто знает, были ли когда-нибудь инциденты с микрометеоритами на орбите? Если были, то с какими последствиями? Чем закончилось?
Аноним 07/06/16 Втр 21:11:01  298617
Возник вопрос, навеянный этими роликами.
https://www.youtube.com/watch?v=p8zPx41oxwE
https://www.youtube.com/watch?v=eyNhhRCCMiI

Если вещество в черной дыре сжато в одну точку, то что произойдёт, если точка будет настолько плотная, что начнёт выворачиваться через саму себя наизнанку и станет расширяться в обратную сторону? Если я правильно понимаю, то такой трюк невозможен даже в теории?
Аноним 07/06/16 Втр 21:14:53  298618
>>298616
Были. В обшивке МКС нашли дырочку, но не сквозную.
Аноним 07/06/16 Втр 21:20:44  298619
(12Кб, 700x500)
>>298618
А еще в Куполе совсем недавно скол нашли.
Аноним 07/06/16 Втр 21:23:38  298620
Почему космонавты и ракеты не улетают куда-то(хуйзнаеткуда) впизду когда они выходят на орбиту?
аааа... Земное притяжение?
А когда вне орбиты почему никуда не сдувает?
Ведь мы летим куда-то, точнее вся наша солнечная система.
Аноним 07/06/16 Втр 21:25:57  298621
>>298620
> А когда вне орбиты почему никуда не сдувает?
> Земное притяжение?
Бинго! А дальше будет притяжение Солнца, Галактики, Аттрактора. А в конце концов они улетят впизду, да.
Аноним 07/06/16 Втр 21:28:09  298622
>>298621
Ох лол. Т.е. нас притягивает какая-то ебучая ебала здоровая? А потом просто распидарасит? Ну... как потом, через миллиарды или больше земных лет?
Аноним 07/06/16 Втр 21:28:29  298623
>>298621
Дарья, eto ti?
Аноним 07/06/16 Втр 21:38:37  298626
>>298622
>Ох лол. Т.е. нас притягивает какая-то ебучая ебала здоровая? А потом просто распидарасит? Ну... как потом, через миллиарды или больше земных лет?
Всё же предполагают, что расширение пространства распидорасит все силы (включая гравитацию), и все улетят в ебеня. А там и их распидорасит.
Аноним 07/06/16 Втр 21:39:43  298628
>>298626
Что значит расширение пространства? Расширение вселенной?
Аноним 07/06/16 Втр 21:39:51  298629
>>298622
> Т.е. нас притягивает какая-то ебучая ебала здоровая?
Земляшка-то? Да.
> А потом просто распидарасит?
Ну, как распидорасит. Вон, Земля уже миллиарда с четыре крутится вокруг солнца, и пока не распидорасилась (ну, если не считать Луны, но это не точно). Кеплеровские орбиты позволяют довольно долго крутиться вокруг более здоровой ебалы, пока либо не затормозишься об атмосферу, либо не затормозишься излучаемыми гравитационными волнами, либо не прилетит какая-нибудь хуйня со стороны и не выкинет тебя нахуй с насиженного места уже своим притяжением. Ну или таки случится Великий Разрыв, и впизду улетит вообще все, даже небо и Аллах развалятся на кварки.
Аноним 07/06/16 Втр 21:43:01  298630
>>298629
>Т.е. нас притягивает какая-то ебучая ебала здоровая? Земляшка-то? Да
Чё ёбнутый?
Я не про притяжение солнца и земли, а о притяжении другом. Вот наша солнечная ситстема летит куда-то, её что-то притягивает? Я про это притяжение.
Аноним 07/06/16 Втр 21:55:12  298633
>>298630
> Чё ёбнутый?
Да нет, в самый раз.
> Вот наша солнечная ситстема летит куда-то, её что-то притягивает? Я про это притяжение.
Ну вот смотри. Кроме нашего Солнца рядом с нами дохуя-дохуя еще других звезд. Все вместе называются галактикой. Все они притягивают друг друга, каждая притягивает каждую (и все вместе крутятся, потому что если в космосе что-то не крутится, это значит что оно ещё не успело раскрутиться). Но если посмотреть наружу галактического диска (предположим для простоты, что звезды по нему распределены равномерно. На самом деле, в центре их в среднем больше, но это только усилит эффект), мы, находящиеся не в его центре, увидим меньше звезд, чем если посмотрим на центр (потому что там будут еще звезды с другой половины диска). Соответственно, в среднем к центру нас притягивает больше, но притягивает нас не какая-то одна большая хуйня (не считая сверхмассивной черной дыры в центре, но она тут не доминирует), а много звезд поменьше.
Примерно так же друг к другу притягиваются галактики, только слабее, потому что дальше. И где-то там на другой стороне нашего галактического диска есть некая большая-большая хуйня, которая притягивает нашу галактику и кучу соседних. Что это за хуйня, никто 146% точно не знает, потому что из-за своей же галактики её толком не разглядеть.
Аноним 07/06/16 Втр 21:57:47  298634
>>298633
Понял.
Аноним 07/06/16 Втр 21:59:43  298637
Что такое сингулярность или что названо таким названием?
Аноним 07/06/16 Втр 22:08:37  298638
Посоны, как бесплатно скачать приложение Stellaris на мою люмию? Или таки потратить 120 рублей?
Аноним 07/06/16 Втр 22:09:30  298639
>>298628
>Что значит расширение пространства? Расширение вселенной?
Ну да, хотя для меня это немного странная формулировка. Изменяется метрика - растет расстояние от тебя до всех остальных. И рост этот (в нынешнею космологическую эпоху) всё ускоряется.
https://ru.wikipedia.org/wiki/Закон_Хаббла
Аноним 07/06/16 Втр 22:09:41  298640
>>298638
Через ломанный маркет, если андройд.
Аноним 07/06/16 Втр 22:10:54  298642
>>298640
Я ж написал люмию. Есть какие то трюки с изменением региона. Вообще стелларис стоит этого?
Аноним 07/06/16 Втр 22:11:06  298643
>>298640
Или это на комп?
Аноним 07/06/16 Втр 22:11:37  298644
>>298642
Кинь ссылку на игру.
Аноним 07/06/16 Втр 22:13:01  298645
>>298642
Стелларис или стеллариум? Стеллариум самый годный планетарий в маркете, я отжалел на него все-таки. А так, тебе лучше в /mobi/ сходить в виндотред, может там помогут. Но вряд ли, с пиратством вроде туго на винде.
Аноним 07/06/16 Втр 22:16:13  298648
>>298644
Это приложение- карта звездного неба для нюфагов. Венеры и сатурны искать всякие
Аноним 07/06/16 Втр 22:19:07  298651
(116Кб, 480x800)
>>298645
>>298644
Вот такая хуитка
Аноним 07/06/16 Втр 22:20:15  298653
(20Кб, 630x440)
>>298630
>Я не про притяжение солнца и земли, а о притяжении другом. Вот наша солнечная ситстема летит куда-то, её что-то притягивает? Я про это притяжение.
Да, есть такая ебола. Достаточно близко, и локально.
https://ru.wikipedia.org/wiki/Сверхскопление_Шепли

Но и его что-то притягивает. "Узлы" стен (и сами нити и стены) в ячеисто-филаментарной структуре Метагалактики.
Аноним 07/06/16 Втр 22:37:38  298657
(436Кб, 480x800)
Еще вот такую приложуху поставил, ичсх когда приближаешь планету - она адово летит по орбите
Аноним 07/06/16 Втр 22:41:22  298658
>>298637
>Что такое сингулярность или что названо таким названием?
Какие же вы стали ленивые.
https://ru.wikipedia.org/wiki/Гравитационная_сингулярность
Аноним 07/06/16 Втр 23:45:28  298664
(41Кб, 556x420)
(69Кб, 293x212)
>>298616
Полным-полно. Вся МКС например испещрена мелкими попаданиями MMOD (Micrometeoroids and Orbital Debris). Радиаторы например чутка повреждены, СБ, и так далее. Сквозных пробоин конечно не было, по крайней мере пока. А от крупных обломков уворачиваются.

Шаттл ввиду размеров тоже постоянно бомбардировало, например STS-118 пиздануло три раза, один из них в иллюминатор, а радиатор вообще пробило навылет.

Со спутниками тоже бывали проблемы, корорые можно объяснить только внешним ударом. Хотя самое известное столкновение это все же столкновение двух ИСЗ, один из которых к тому времени давно был мусором: https://en.wikipedia.org/wiki/2009_satellite_collision

Алсо, есть такое понятие как микрометеорная эрозия - плавная деградация материалов от столкновения с пылинками. Причем не обязательно механическая - от высвобождения кинетической энергии при столкновении может даже структура материала или химический состав измениться локально.
Аноним 07/06/16 Втр 23:49:43  298666
>>298628
Это значит, когда выдвигали гипотезы, объясняющие красное смещение, собрали все в кучу и выбрали самую хуевую.
Аноним 07/06/16 Втр 23:49:45  298667
>>298617
Точка - не сфера, и куда-то расширяться не может и не будет, вопрос из разряда "что будет, если зеленые слоники полетят на Венеру, махая ушами". алсо, у тебя в ролике выворачивание с самопересечениями, без них это невозможно топологически.
Аноним 07/06/16 Втр 23:56:54  298669
Посоны, помогите найти видео, пожалуйста, гугл и ютуб молчат.
В общем, на видео записан тест какой-то маленькой хуитки с несколькими ракетными двигателями, которая ими коротко плюётся и благодаря охуенной стабилизации на секунд десять-пятнадцать буквально замирает над полом, а потом её ловят в белую сеть.
С меня лучи добра и сотни нефти.
Аноним 08/06/16 Срд 00:19:46  298674
>>298669
Multiple Kill Vehicle. Ближе к /wm/, правда, но так эта хуитка ни во что толковое и не выросла.
https://www.youtube.com/watch?v=KBMU6l6GsdM

Алсо, системы посадки в современных АМС и движки ориентации спутников в импульсных режимах как раз и работают, модулируя тягу ШИМом - в мелкодвижках плеваться по сотне раз в секунду проще, чем ебаться с дросселированием. мелко-ЖРД вообще намного сложнее делать чем средние
Аноним 08/06/16 Срд 00:20:59  298675
>>298669
Врядли это твоя мамка, ее только Сатурн-5 поднимает
Аноним 08/06/16 Срд 00:25:03  298676
(888Кб, 1280x822)
>>298674
Спасибо тебе, анон. Держи няшу.

>>298675
Ваганыч, залогинься.
Аноним 08/06/16 Срд 00:26:54  298677
>>298674
>эта хуитка ни во что толковое и не выросла
Охуел?
Аноним 08/06/16 Срд 01:39:12  298686
>>298677
А во что-то разве выросла? Я думал её забросили нахуй. от систем вооружений я далек
Аноним 08/06/16 Срд 01:45:46  298688
>>298686
Всё, что сейчас в спешном порядке меняют прям в шахтах - суть дериватива от него. Да, теперь это не разводилка болванок.
Олсо, Вики говорит:
>A similar program was restarted in August 2015, when Raytheon was awarded a $9,775,608 contract to test and design a Multi-Object Kill Vehicle concept.
Аноним 08/06/16 Срд 03:35:52  298692
>>298676
Что это такое?
Аноним 08/06/16 Срд 03:37:48  298693
>>298692
Бесполезный манялет. Еще менее реальный и нужный, чем X-20;
Аноним 08/06/16 Срд 04:43:16  298694
>>298633
А я не понял. Я был уверен, что аттрактор в центре галактики. Нарисуйте в пэйнте схематично для дибила.
Аноним 08/06/16 Срд 07:28:11  298695
>>298225 (OP)

1 Почему Кассини и Вояджер смогли преодолеть силу притяжения (простите если это не так называется) Юпитера и не стали вечно кружить вокруг него как его многочисленные луны а полетели дальше.

2 Как именно фотографировались газовые гиганты с Вояджера и с Кассини?Как учёные понимали в какую сторону должна быть направлена камера?

Извините.
Аноним 08/06/16 Срд 08:43:14  298696
>>298694
Вот же, там и аттрактор обозначен: >>298653
Это просто большое скопление галактик с вероятностью 99%.
Аноним 08/06/16 Срд 09:35:09  298698
>>298695
1. Скорость слишком большая, хуяк и проскочил. Юпитер лишь немного отклонил траекторию, попутно увеличив им скорость. Чтобы остаться, надо было пёрнуть назад чем-нибудь во время пролёта. Просто так при пролёте другого тела на орбиту не выйти, по той же причине, по которой снаряд из пушки на планете нельзя вывести на её орбиту. (в идеальном случае, т.е. не считая баллистического захвата и прочих хитростей с парусами и внешними возмущениями)

2. Построили систему координат Юпитера, смоделировали траекторию аппарата, всё это обычная баллистика, траектории хорошо предсказуемы. Но как я понимаю, тебя больше интересует как Вояджер знал, куда поворачиваться? Ну у него были солнечный и звездный датчики (подсистема AACS). Сначала находился самый яркий источник на небе, это солнце. Дальше борткомпьютер строил маневр для двигателей (Вояджеры не использовали маховики ориентации) и поворачивался одним концом к нему. Потом аппарат закручивался вокруг вектора на солнце и ждал пока рисунок на звездной камере не совпадет с заложенным шаблоном. Всё, базовая ориентация построена, теперь от калиброванного эталона можно отклоняться в любую сторону для связи с Землёй или наводить камеру и инструменты согласно полётному заданию - вместе с показаниями бортовых часов и гироскопа аппарат будет точно знать куда смотрит в данный момент. Процедуру можно повторить при необходимости, и время от времени нужно, т.к. гироскоп со временем начинает пиздить.
Аноним 08/06/16 Срд 09:38:01  298699
Видимо задать этот вопрос надо сюда.
Почему, зачем, за что и как выпилили КСП?
Аноним 08/06/16 Срд 10:05:51  298704
>>298699
>Почему, зачем, за что
> в том, что в последние полгода-год тред стал очень слабо соответствовать тематике раздела, превращая /spc/ в маленький филиал /vg/. Уже как два месяца на него создавались жалобы в /d/ от анона из раздела, да и сама обстановка в треде мало радует. И за его пределами, потому что многие, кто посещает огурцетред и постит в нём, откровенно и напрямую гадят политотой за пределами треда.

>как
>По этим причинам было решено перевести тред в /vg/, под тэг /ksp/. Тем более вы сами создали себе перекат туда, и он даже умеренно жив
Аноним 08/06/16 Срд 10:13:49  298705
>>298699
Это вопрос для /d/, где долго срались на эту тему. Краткий ответ - моча обезумела, иди в /vg/.
Аноним 08/06/16 Срд 10:37:11  298706
>>298667
А точка же была сферой до того, как сжаться в точку. Мне интересно, что получится, если допустить перегибы Тогда точка сможет пройти сквозь саму себя и что получится? И в каком измерении такое может быть?
Аноним 08/06/16 Срд 10:38:10  298707
>>298698
Очень хорошо! Спасибо большое!
Аноним 08/06/16 Срд 10:38:30  298708
>>298699
Мочерне не понравилось, что тред набрал популярность на слоудоске, которую они считают не доской для свободного общения увлеченных космосом людей, а собственным илитным форумом. Им стало неприятно, что огуречные треды начали быстро смывать такие охуенно важные темы, как анимублядский_гет-тред, тред про навозных жуков, и треды достигнувшие бамплимита. Вместо того, чтобы заморачиваться удалять старые бамплимитные ксп-треды или дать кодеру идею написать под это дело простейший скрипт, мочерня за всех решила пидорнуть ксп-треды в парашу для общения двоечников после тяжелого дня в школе. Сделала она это под предлогом якобы нерелейтеда - это самолеты, вертолеты и корабли не каноничной конструкции, которые частенько крафтили огурцы. Естественно это надуманный повод, поскольку если придраться к этому то многие другие треды можно пидорнуть вон, самолет это та же ракета, только с крылышками, как буран или шатл и космонавтами ирл обычно становятся летчики, вертолеты тоже используют ирл для транспортировки частей космических кораблей и спасения экипажа, а не каноничная конструкция это полет фантазии, что бывает хорошо при поиске новых путей. Еще они обманули Абу, который очень редко бывает в спейсаче и не увлекается космосом, сказав ему якобы от нас есть дохуя жалоб на нерелейтед и мы в большинстве согласны с этим их решением.
Аноним 08/06/16 Срд 11:10:51  298711
>>298708
Но так и есть, эта хуйня выбешивала всех адекватных спейсанов, выросших из возраста школоблядей. И пидорнули ее не из-за самолетов вертолетов. А потому-что это ИГРА, а место играм в /vg.
Аноним 08/06/16 Срд 11:18:29  298712
>>298711
Пiшов нахiй. Спейсач всегда был загоном для аутистов потому и ламповый. В огурцетредах сидели не столько игродауны, сколько аутисты, которым доставляет космотематика. Оттуда они частенько забредали в треды тупых вопросов, роскосый треды и прочие, создавая боле-менее оживленное общение. В конце концов огурцы - это дрочильня-популяризатор науки. И какой-нибудь ньюфаг, клепавший ракеты/самолетики, авось да и задастся различными вопросами, отписавшись в релейтед тредах. С выгоном огурцов на мороз на спейсаче осталось уже даже не 3.5, а 1.5 анония, что на пользу спейсачу явно не пошло.
Аноним 08/06/16 Срд 11:30:01  298713
>>298712
>>298708
Двачую. В огурцов не играл, но подтверждаю ебанутость тупорылой мочезаглотины, устраивающей личные форумчики с анальными неписанными правилами, а не свободное общение.
> С выгоном огурцов на мороз на спейсаче осталось уже даже не 3.5, а 1.5 анония, что на пользу спейсачу явно не пошло.
На спейсаче живо полтора треда - SpaceX из-за частых запусков каждые 2-3-4 недели, поэтому всегда есть что обсуждать и унылый батутный тралленг Роскосмоса.
Аноним 08/06/16 Срд 11:45:43  298715
>>298706
А потом стала точкой, потеряв топологические свойства. На самом деле, ответ простой - никто не знает что там происходит внутри сингулярности, она реально точка для известной нам физики.
Аноним 08/06/16 Срд 12:10:16  298717
>>298708
Помнится какой-то анально уязвленный вечно визжал про то что фильмотред надо пидорнуть, потом огурцы, периодические визги в сторону SE и космоарт-треда. Это один ебанутый или их тут несколько?
Мочерня совершенно неадекватна, плодит сторуменов и вертит местных аутистов на хую запиливая свою персональную доску.
Инжинегры на месте https://2ch.hk/spc/res/291233.html а огурцов пидорнули в школопарашу. Ясен пень, Абу не разберет какая между ними разница и какого хуя нам не нравится.
Аноним 08/06/16 Срд 12:38:41  298720
>>298390
Смотря как "помещать", если медленно снижать давление до нуля - баротравмы не будет.
Аноним 08/06/16 Срд 13:22:25  298722
(4Кб, 254x199)
Кто-нибудь знает сколько deltav было у этой поделки?
Аноним 08/06/16 Срд 14:05:29  298725
>>298711
> А потому-что это ИГРА, а место играм в /vg.
Поссали всем SE тредом на тебя.
Аноним 08/06/16 Срд 14:34:24  298727
>>298722
Какая разница, если она в атмосфере летала?
Аноним 08/06/16 Срд 14:45:03  298728
>>298727
> овер 100км
> в атмосфере
Аноним 08/06/16 Срд 15:41:33  298731
>>298728
Мы вообще летаем в атмосфере Солнца, так-то.
Аноним 08/06/16 Срд 15:48:56  298733
>>298722
2.4км/с.
Аноним 08/06/16 Срд 16:36:47  298736
>>298728
Атмосфера земли простирается на 1000км + ещё 1 тысяча на суборбитальные атомы.
Аноним 08/06/16 Срд 16:43:01  298740
(17Кб, 640x480)
>>298736
>Атмосфера земли простирается на 1000км + ещё 1 тысяча на суборбитальные атомы.
До Облака Оорта, если быть точным. Плотность атмосферы Земли там очень невелика, но она есть.
Аноним 08/06/16 Срд 16:46:20  298742
>>298736
На 1500 примерно, или около.

>>298740
>До Облака Оорта, если быть точным. Плотность атмосферы Земли там очень невелика, но она есть.
Нихуя. На определенной высоте состав среды уравнивается с межпланетной.
Аноним 08/06/16 Срд 16:49:11  298745
>>298742
>Нихуя. На определенной высоте состав среды уравнивается с межпланетной.
Т.е. в Облаке Оорта нет ни единого атома вещества из атмосферы Земли?
Аноним 08/06/16 Срд 16:50:23  298746
>>298745
Хм, ну разве что если так, тогда да. и то вряд ли
Аноним 08/06/16 Срд 16:59:31  298749
>>298225 (OP)
Хелп аноны, есть 2 вопроса на которые я уже давно хочу узнать ответ.

1) На сколько я знаю, в первую секунду после большого взрыва вселенная уже стала больше, чем видимая сегодня. Вопрос: как частицы и материя вообще могла так быстро разнестись по вселенной, если максимальная скорость не может превышать скорость света?

2) НАСА не так давно говорили, что нашли на Марсе жидкую воду, что она там какие-то русла оставляет и т.д. Даже фотки тогда показывали. Но как вообще на Марсе может быть жидкая вода если там почти нет атмосферы? То есть, по идее на Марсе у воды не должно быть жидкого состояния и она должна изо льда сразу превращаться в пар.
Аноним 08/06/16 Срд 17:13:59  298753
>>298749
>как частицы и материя вообще могла так быстро разнестись по вселенной, если максимальная скорость не может превышать скорость света?
Опять платину толкаешь, шакал? Скорость перемещения материи ограничена некой фундаментальной константой c, да. Но на скорость расширения самого пространства ОТО не накладывает никаких ограничений.
Аноним 08/06/16 Срд 17:19:25  298755
>>298753
Платина? В смысле это вопрос, который постоянно задают?
Но спасибо за ответ.
Аноним 08/06/16 Срд 17:20:02  298756
>>298749
1) нельзя сказать как она расширилась. Ввиду того что это не взрыв в привычном понимании. Он как бы взорвался во всех точках сразу. То что 13 млрд это только свет успел прилететь, расширение же это 13 млрд расширенные в 40 с хвостиком.

2) Там есть низины с давлением и создаются условия для воды.
Аноним 08/06/16 Срд 17:22:32  298757
>>298756
Интересно. Спасибо.
Аноним 08/06/16 Срд 18:26:23  298765
(338Кб, 1689x700)
(250Кб, 1374x714)
>>298749

Вот этого долбоёба >>298756 про воду не слушай.

Вода на Марсе существует в жидком виде в основном в теплые месяцы, а жиденькой она остается благодаря тому, что это не чистая вода, а адовый раствор перхлоратов. Это как соленая вода не замерзает при нуле градусов, так и там из-за этих солей она имеет более низкую температуру замерзания, и может существовать в жидком виде, при низком давлении.
Аноним 08/06/16 Срд 18:29:18  298766
Из треда в тред пощу это.
Быстрее света нельзя только передать информацию, все что не нарушает этого правила возможно.
Аноним 08/06/16 Срд 18:37:51  298767
>>298766
Строим длинный стержень, допустим длиной в световую минуту, из сверхлёгкого и сверхпрочного материала, присоединяем к ротору шагового электродвигателя. Включаем двигатель, стержень делает оборот за долю секунды, информация передана быстрее скорости света.
Аноним 08/06/16 Срд 18:54:41  298768
>>298765
Какая, кстати, самая высокая температура поверхности зарегистрирована на Марсе? Где-то попадалось, что иногда и за 0 Цельсия переваливает.
Аноним 08/06/16 Срд 18:55:15  298770
>>298767
Как вы заебали, мамкины физики. Одно и тоже уже несколько лет. Скорость взаимодействия между молекулами ограничено. Так что как ты не ебись со стержнем с одной стороны, на другой об этом узнают не быстрее с.
Аноним 08/06/16 Срд 18:56:00  298771
>>298768
На экваторе в летний погожий день и плюс 15 бывает. А поверхность и больше нагревается.
Аноним 08/06/16 Срд 19:17:03  298773
>>298767
Стержень не делает оборот за долю секунды. Импульс(информацию) нужно передать, а быстрее скорости света её передать нельзя. пойдет волна импульса, при том настолько медленно, что её можно глазами будет увидеть.

И с той же оперы, например. Посвети огромным и мощным фонарем в луну, поставь палец перед фонарем и пошевели им. Тень пройдет по луне за долю секунды, даже, казалось бы, быстрее скорости света. Но никакой информации между крайними точками тени передано не было.
Аноним 08/06/16 Срд 19:28:56  298774
>>298766
>Светом или любым другим электромагнитным явлением нельзя передать информацию быстрее света в локальных условиях.
Поправил тебя, не блогодори.
Аноним 08/06/16 Срд 19:36:25  298775
>>298774
Говно поправка. Что, пальцем значит передавать можно?
Аноним 08/06/16 Срд 19:54:28  298778
>>298775
Если разогнать его больше скорости света, то вполне.
Аноним 08/06/16 Срд 19:56:40  298779
>>298664
Воу, спасибо!
Аноним 08/06/16 Срд 20:15:49  298781
(945Кб, 738x966)
Кто на фото? Какой год?
Аноним 08/06/16 Срд 20:18:36  298783
>>298778
Совсем мальчик поехал.
Аноним 08/06/16 Срд 20:21:04  298785
>>298225 (OP)
Спейсаны, поясните, пожалуйста, за спутники связи. Сколько их всего, какие виды связи они обеспечивают? Иридиум же не единственные подобные спутники?
Аноним 08/06/16 Срд 20:35:40  298789
>>298765
>благодаря тому, что это не чистая вода, а адовый раствор
Что не отменяет того факта, что и благодаря низменному давлению.
Аноним 08/06/16 Срд 20:58:39  298790
>>298789
>что и благодаря низменному давлению
Пиздуй нахуй со своими охуительными историями, дебила кусок.
Аноним 08/06/16 Срд 21:25:51  298791
>>298781
Леонид Денисович Кизим.
Фото скорее всего 1984 года сделано во время экспедиции Союз Т-10, на Салют-7 так как эта фотка красуется на обложке книги "Космонавтика СССР" 1986 года издания
Аноним 08/06/16 Срд 23:21:48  298797
>>298736
Аааа. То есть почти все летают в атмосфере?
Аноним 08/06/16 Срд 23:23:16  298798
>>298790
Видимо мы говорим о разных вещах.
Аноним 08/06/16 Срд 23:24:13  298799
>>298781
А он не обгорел?
Аноним 08/06/16 Срд 23:43:48  298802
>>298798
Я говорю о том, что ты нихуя не знаешь и кукарекаешь.
Аноним 09/06/16 Чтв 00:10:03  298806
>>298802
Вот долбоеб
Аноним 09/06/16 Чтв 00:18:15  298807
>>298806
Съеби нахуй отсюда и не вякай, если не владеешь предметом.
Аноним 09/06/16 Чтв 00:20:49  298808
Низменное, блядь, давление.
Спасибо, посоны, весь вечер проигрываю.
Аноним 09/06/16 Чтв 00:33:34  298811
>>298808
Давление в низине

А ты кретин
Аноним 09/06/16 Чтв 00:35:58  298813
>>298811
Иди нахуй, низменный.
Аноним 09/06/16 Чтв 00:40:29  298815
>>298813
> покпокпоккудах
Аноним 09/06/16 Чтв 01:06:43  298821
>>298811
В какой низине, ебнутый ты пидорас?
У тебя Феникс в НИЗИНЕ стоял, даун?

Там потеки воды на склонах ебучих ГОР стекают сверху вниз. Съеби нахуй отсюда, не беси меня.
Аноним 09/06/16 Чтв 01:42:36  298830
(1250Кб, 4000x4328)
>>298778
Аноним 09/06/16 Чтв 01:47:39  298831
(7959Кб, 640x360, 00:01:21)
Че они пиздят? Ведь тогда бы сс была воронкообразная.
Аноним 09/06/16 Чтв 01:50:55  298832
>>298831
Шел 52 тред, ньюфаги продолжают таскать эту видюху и требовать разъяснений.
Хоть какая-то стабильность.
Аноним 09/06/16 Чтв 01:56:00  298833
>>298832
И к чему пришли в этих 53 тредах?
Аноним 09/06/16 Чтв 02:01:59  298834
>>298833
К тому, что не плохо бы астрономию в школы вернуть. Глядишь и очевидные вещи, выдаваемые за невероятные открытия, не будут шокировать обывателя.
Например вполне обыденным фактом является то, что во Вселенной все куда-нибудь да движется, а куда и как сильно зависит от системы отсчета.
Аноним 09/06/16 Чтв 02:05:52  298835
>>298834
А если убрать ВСЕ ориентиры, можно узнать движешься или стоишь?
Аноним 09/06/16 Чтв 02:10:25  298836
>>298835
Если совсем все, и движение равноускоренное и прямолинейное, то нет.
Аноним 09/06/16 Чтв 02:28:31  298838
>>298836
То есть не имеет значения, движешься или стоишь?
Аноним 09/06/16 Чтв 02:36:45  298839
>>298838
Ты сейчас сидишь на жопе, или идешь куда-то?
Аноним 09/06/16 Чтв 02:41:25  298840
>>298839
Сижу.
Аноним 09/06/16 Чтв 02:45:55  298841

>>298838
Вот вроде ты сидишь на жопе и никуда не движешься. Но ты сидишь на поверхности шарика, который крутится со скоростью более 1000 км\час.
Можешь почувствовать это?
Если взять за точку отсчета Солнце, то ты летишь со скоростью 30 км\секунду. Можешь почувствовать?
А если центр Галактики, то со скорость. 225 км\сек. Можешь почувствовать?
А в направлении скопления галактик в созвездии Девы ты движешься со скоростью 400 км\сек.
Можешь почувствовать?

Имеет ли все это значение, кроме того, что ты сидишь на жопе и никуда не движешься?
Аноним 09/06/16 Чтв 03:29:34  298842
>>298838
>То есть не имеет значения, движешься или стоишь?
Это Первый закон Ньютона, педрила.
Аноним 09/06/16 Чтв 03:37:55  298843
>>298696
А где тут тогда предполагаемая черная дыра?
Аноним 09/06/16 Чтв 03:54:20  298846
>>298831
С чего бы это ей быть воронкой, если планеты были образованы вместе с Солнцем и получили ровно тот же момент импульса с самого своего зарождения. Кольца Урана вполне себе аналогичный пример. Но видео действительно лишь "красивка" для научпоп быдлеца из пабликов. Плоскость эклиптики к галактической плоскости вращения наклонена на ~60 градусов.
Аноним 09/06/16 Чтв 04:05:38  298847
Но ведь на плонеты и прочий мусор больше воздействует гравитация солнца, нежели первоначальный импульс. По большому счету они летят в солнце, которое летит в пространстве. Должна воронка быть.
Аноним 09/06/16 Чтв 04:46:53  298848
>>298831
Как долбаебы клюете на этот фейк.
Аноним 09/06/16 Чтв 08:01:59  298849
>>298848
Ну, не совсем фэйк. Оно так и есть. Просто нихуя не сенсация.
Аноним 09/06/16 Чтв 09:50:16  298866
>>298843
Какая нахуй предполагаемая чёрная дыра? Кем предполагаемая? Тобой? Обколются своим Рен-ТВ и ебут в спейсаче мозги.
Аноним 09/06/16 Чтв 09:56:09  298867
>>298849
Оно не так и есть, солнце летит не строго вниз, относительно плоскости диска Галактики, а ещё по диску Галактики, вокруг центра. Так что это в любом случае маняхуйня.
Аноним 09/06/16 Чтв 10:56:31  298887
>>298867
Наклонение Солнечной системы к плоскости галактики 60 градусов.
Аноним 09/06/16 Чтв 10:58:07  298888
>>298887
И че?
Аноним 09/06/16 Чтв 10:59:44  298889
>>298888
А то, что на симуляции Солнце летит не вниз.
Аноним 09/06/16 Чтв 11:13:45  298891
>>298889
На этой, как ты выразился, симуляции, оно вообще хуй знает куда летит. Никаких привязок к чему либо я там не наблюдаю.
Аноним 09/06/16 Чтв 11:17:34  298892
>>298891
Летит вокруг центра галактики, плоскость системы наклонена.
Аноним 09/06/16 Чтв 11:20:36  298893
>>298892
Где там ты увидел центр Галактики, где там указано наклонение? Алсо, наклонение ещё ничего не значит Земля тоже имеет наклонение, к плоскости эклиптики и че?
Аноним 09/06/16 Чтв 11:30:26  298895
>>298835
Конечно можно.
Аноним 09/06/16 Чтв 12:39:35  298901
>>298893
>Земля тоже имеет наклонение, к плоскости эклиптики
Просто иди на хуй.
Аноним 09/06/16 Чтв 12:46:40  298903
(23Кб, 1230x770)
Заебали.
Аноним 09/06/16 Чтв 12:51:52  298905
>>298903
60 градусов — это почти прямой угол, а ты уж совсем маленький нарисовал.
Аноним 09/06/16 Чтв 12:56:24  298908
>>298892
На видео нет никакого наклона. Плоскость эклиптики на нем очевидно перпендикулярна орбите Солнца. В реальности не так.
Аноним 09/06/16 Чтв 12:57:48  298909
(24Кб, 1230x770)
>>298905
60 градусов это далеко не прямой угол.
Аноним 09/06/16 Чтв 12:58:04  298910
(509Кб, 1399x712)
Аноним 09/06/16 Чтв 13:04:47  298911
>>298835
Еще достаточно точно можно определить движение по дипольной анизотропии реликтового излучения.
Аноним 09/06/16 Чтв 13:12:59  298912
>>298908
На видео не перпендикулярна, а примерно 60 градусов. Это можно увидеть проследив за Юпитером на кадрах с 0:29. Он то совсем за Солнцем, то чуть перед ним.
Аноним 09/06/16 Чтв 13:15:29  298913
>>298911
>убрать ВСЕ ориентиры
Аноним 09/06/16 Чтв 13:53:02  298917
>>298912
>а примерно 60 градусов
Хах, хахах, нахуй иди! Последил я за Юпитером и увидел, что он дальше Сатурна крутится. Ну это уже полный пиздец.
Аноним 09/06/16 Чтв 14:05:05  298919
>>298917
Как это относится к обсуждаемому наклонению?
Аноним 09/06/16 Чтв 14:19:32  298920
>>298912
Нет, там всё так же 90 градусов.
Аноним 09/06/16 Чтв 14:28:22  298922
>>298909
Хм, так тоже получатся спирали.
Аноним 09/06/16 Чтв 15:07:26  298925
Кто уронил Уран набок?
Аноним 09/06/16 Чтв 15:12:56  298926
Почему не построят станцию на орбите луны? Рядом же
Аноним 09/06/16 Чтв 15:46:16  298932
>>298926
Далеко. Для человека долгое пребывание выше 400 км, сложно и с технической т.з. и с т.з возможностей. Сейчас предел это МКС, может что повыше потом запилят, но не в ближайшие 2 десятилетия.
1 очередь нет ракет для доставки не на низкую орбиту.
2 Выше и радиации больше, а уж тем более на луне, читай межпланетарная среда.
3 Космонавты не нежить им есть надо => или запускай каждые 2 недели сухпай, либо делай космоферму в несколько гектаров. Ни то ни другое сегодня невозможно. Кислород то же самое.
4.Денег нет, кризис кругом.
5.Задачи есть, но они так специфично-перспективны, что в этом веке никому не интересны. Помним золотое правило современной экономики: "деньги ради денег в кратчайшие сроки".
Это в основном касаемо любой станции выше 400 км не говоря о 384000км
Аноним 09/06/16 Чтв 15:52:17  298934
>>298922
Не совсем. если глядеть снизу то будет момент когда земля опережает солнце на 74500000 км
Аноним 09/06/16 Чтв 16:04:31  298935
Поясните на простом, что такое эклиптика.
Аноним 09/06/16 Чтв 16:43:57  298943
>>298935
Это плоскость орбиты Земли.
Аноним 09/06/16 Чтв 16:48:31  298946
>>298225 (OP)
Отлично, начались каникулы и поэтому мудила школьник-кун выходит на связь.

Суть моего вопроса: Если сделать себе стратостат, то есть возможность с него закинуть в космос свой самодельный спутник? Ясен хуй что нельзя, иначе бы не тратили миллиарды на строительство огромных фалосов. Так вот, в чем проблема закинуть с такого стратостата спутник на орбиту? Ясен хуй, что это все гуглится, наверное в пять минут, но мне лень, надеюсь на доброго анона, который все разжует.
Аноним 09/06/16 Чтв 17:44:16  298952
>>298946
Соси хуй, раковый.
Аноним 09/06/16 Чтв 17:52:44  298954
>>298952
Ух ты, мистер илитарии, как вам на двачах живется?
Аноним 09/06/16 Чтв 18:25:20  298956
>>298946
Теоретически возможно, но нужен слишком большой аэростат (размеры растут с высотой и нагрузкой чрезвычайно быстро), сложная система старта и все это ради весьма малой выгоды.

>>298954
Констатирую, что старт с аэростата - слишком тупая и платиновая тема даже для этого треда. Да, даже из треда тупых вопросов могут послать в биореактор иногда.
Аноним 09/06/16 Чтв 18:27:47  298957
>>298925
Сатурн по пьяни, было дело.
Аноним 09/06/16 Чтв 18:27:56  298958
>>298946
Ракеты нужны не для подъёма, а для 1 космической. т.е. тебе надо запустить на дирижабле ракету не на много легче, а то и больше по весу, ввиду того, что это будет не протон, а новая ракета под дирижабль, которую требуется разрабатывать десятилетиями.
Аноним 09/06/16 Чтв 18:31:42  298959
>>298926
Его >>298932 не слушай. Окололунная станция сейчас представляется логичным продолжением околоземных, во всех агентствах. Как в плане перевалочного пункта к Луне и Марсу, так и в плане развития технологий автономности пребывания в космосе. Скорее всего - на удалённой ретроградной орбите (спиралевидной, очень высокой, очень стабильной). Так что может и будет.
Аноним 09/06/16 Чтв 18:32:13  298960
В чём профит РК Зенит, помимо красивого названия?
Аноним 09/06/16 Чтв 18:42:09  298962
>>298919
Почему наклонение вообще осуждается в контексте собственного движения Солнца, а?
Аноним 09/06/16 Чтв 18:44:56  298963
>>298959
Да рассматривается. Только одной хотелкой это не осуществить. Или это из разряда: " через н-лет изобретут ёба материал и заживём"?
Пускай я поверю, что роскосый в 20х полетит на луну, но что бы пусть и группа стран сразу лунную базу сделали. Быть может после МКС мы полетим куда нибудь куда нибудь на 1-2,5к км но не более.
Аноним 09/06/16 Чтв 19:01:01  298964
>>298963
>Или это из разряда: " через н-лет изобретут ёба материал и заживём"?
Это из разряда "новое поколение станций". Корабли, способные достать до лунной орбиты, через несколько лет уже будут летать. Сборка пассивных модулей отработана на МКС, плюс масса новых наработок, в частности роботизированные манипуляторы и легко заменяемые модули для неограниченного продления срока эксплуатации. Еще трансформируемые возможно подтянутся, они еще сократят стоимость замены и поднимут жилой объем. По повышенной автономности, чтобы сократить число технических рейсов, наработки были и есть. Плюс снижение объема наземной поддержки.

В этом ничего принципиально сложного/невозможного нет. У тебя такое тонкое "не жили бохато, неча и начинать" сейчас. На практике никогда готовых технологий не было, при создании каждой новой большой станции, Мир или МКС, решали какую-нибудь качественно новую задачу, для которой в свое время находились такие же отговорки, но при этом были все наработки в сыром виде. И деградации СБ боялись, и невозможности делать быстрые ВКД, и автоколебаний конструкции, и чего только не боялись. Диды не сделали, значит и у вас ничего не выйдет, дурачки. А МКС тем временем крутится себе и крутится, кто мог представить, что банальный ФГБ превратится в такую йобу. Собственно, такие проекты и нужны, чтобы превращать наработки и идеи в готовые технологии. Ну можно сидеть и нихуя не делать, как вариант. А новую станцию на НОО никто делать не будет, МКС исчерпала себя, её прямые функции можно выполнять гораздо менее затратными методами.
Аноним 09/06/16 Чтв 19:03:14  298965
>>298963
> Быть может после МКС мы полетим куда нибудь куда нибудь на 1-2,5к км но не более.
На 1-2.5 тыс км уже радиационные пояса, а принципиальной разницы между 400 и 2500 никакой нет. Там нет смысла делать станцию.

>роскосый
Никто про него и не говорит. Международный проект, вполне осиляемо, МКС же осилили, и шаттлы к Миру.
Аноним 09/06/16 Чтв 19:03:48  298966
>>298956
>>298958
Спасибо за ответы, добра вам, впредь буду знать, что здесь это платиновый вопрос и с ним могут послать нахер. Ну я думаю мне простительно, я тут не особо сижу.
Аноним 09/06/16 Чтв 19:03:56  298967
>>298964
>Корабли, способные достать до лунной орбиты, через несколько лет уже будут летать.
Корабли, способные достать до лунной орбиты, летали еще в 70-х вполне успешно. Толку то от этого, один хуй - дорого пиздец.
Аноним 09/06/16 Чтв 19:05:59  298968
>>298967
Имеется в виду что на данный момент их нет.

>дорого пиздец
Нет. Аполлон был дорогим не потому что мог достать до Луны, а потому что это был Аполлон. Алсо, дороже полёт = меньше рейсов. Не вижу проблемы, задача тренировать автономность в этом и состоит.

Надо решать качественно новые задачи, нет смысла делать одно и то же, это трата денег впустую.
Аноним 09/06/16 Чтв 19:38:50  298971
>>298964
Всё что ты написал правда, только не в это и не в следующее десятилетие. Ладно защита от радиации, придумают, ладно воздух и водамоча. Как ты хавку на орбите вырастишь без наработок? Или сверхтяжи под деревьями растут?
Аноним 09/06/16 Чтв 19:51:40  298972
(98Кб, 640x510)
>>298968
>Аполлон был дорогим не потому что мог достать до Луны, а потому что это был Аполлон.
Две бочки спирта этому ракетчику.
Даже в спейсаче такая простая вещь не до всех доходит.
Аноним 09/06/16 Чтв 20:03:32  298973
>>298967
Не пойму, что тут сверхдорого? Это же не лунная экспедиция с посадкой и возвращением. Тупо закинуть на высокую орбиту и пусть там болтаются. Грузы и человеков возить? Но ведь основные затраты топлива это вывод на ноо. От ноо до высокой лунной орбиты не думаю что настолько дохуя дельты нужно.
Все равно нужно делать шаг вперед, мкс беззадачная хуита, ее держат лишь чтобы окончательно не проебать все полимеры
Аноним 09/06/16 Чтв 20:11:21  298974
А че там с радиацей на орбите? Почему не обматают станцию свинцовой фольгой или чем-нибудь типа этого?
Аноним 09/06/16 Чтв 20:16:43  298975
>>298974
Я думаю, на этой доске пора уже банить за вопросы про радиацию.
Аноним 09/06/16 Чтв 20:26:01  298980
>>298975
Вместо того, чтобы не обратить внимание на этот вопрос, ответить, или же просто запилить фак раз дохуя умный, ты просто начал вонять. Съеби отсюда, человек-пидор
Аноним 09/06/16 Чтв 20:55:25  298997
>>298974
Есть такая штука как наведённая радиация. Так что не спасёт А на МКС суммарная радиация меньше чем при рентгене и это за пол года. У аполлонов в разы больше и уже не похиковать не получится. Полгода может и можно если очень нужно, но год уже смертельно опасно.
Аноним 09/06/16 Чтв 21:18:04  299010
>>298997
И ничего с этим не сделать? Только магнитное поле?
Аноним 09/06/16 Чтв 21:34:33  299020
>>298997
Как тогда планируют пилотируемые миссии на марс? Смертников отправлять будут?
Аноним 09/06/16 Чтв 21:37:41  299023
(234Кб, 485x630)
>>299010
Да в этом-то и одна из основных проблем космонавтики. Мы не можем полноценно находиться в космосе из-за колоссального объема радиации, который получит человек. Пока никто людей на Марс не отправляет, но проблема важная. Решим проблему с радиацией - и сможем спокойно посылать космонавтов. Либо будем посылать на очень короткий срок, потому что это просто смертельно опасно для космонавтов.
Аноним 09/06/16 Чтв 21:38:39  299024
>>299023
Анус свой реши, платиновый рак.
Аноним 09/06/16 Чтв 21:46:56  299025
>>298974
>Почему не обматают станцию свинцовой фольгой или чем-нибудь типа этого?
Потому что это тяжело.

>>299020
Идей масса, в частности пустить воду в переборки, либо применять легкие пенообразующие пластмассы с малой прозрачностью для гаммы. Пока что ни одна не реализована, естественно.

>>299023
Кроме радиации, проблем куча.
Аноним 09/06/16 Чтв 21:48:22  299026
>>299025
Конечно куча. Но это серьёзная преграда на пути создания колоний в космосе.
Аноним 09/06/16 Чтв 21:48:29  299027
>>298971
>Как ты хавку на орбите вырастишь без наработок?
Никто же не говорит о 100% замкнутом цикле, его и не выйдет. Даже наполовину по биомассе замкнуть, и будет заебись.
Аноним 09/06/16 Чтв 22:50:40  299087
Посоны.
Поеду шляться по Европке в скором времени.
Очень упарываюсь космической техникой. Накидайте годных музеев, что ли.
А то там одни нудные самолетики.
Аноним 09/06/16 Чтв 22:51:04  299088
>>299087
Инфы маловато, в англоязычном секторе тоже, да.
Аноним 09/06/16 Чтв 22:54:22  299090
>>299087
В Тулузе емнип был годный музей.
Аноним 09/06/16 Чтв 22:55:48  299091
>>299087
В европке мало годных космомузеев. В Ле-Бурже почуть есть, да в гермашке вроде что-то, разве что. Основные космомузеи в россии (московский и калужский ГМИК + музеи при КБ и институтах) и в штатах (там много всяких).
Аноним 09/06/16 Чтв 23:03:36  299092
>>299090
Вот именно во Францию не буду заворачивать вообще никак.

>>299091
>В европке мало годных космомузеев
А в Штаты дохуя дорого ехать, да.
Аноним 09/06/16 Чтв 23:10:05  299093
>>299027
Давай по грубой прикидке. В день человеконафт съедает по 0.5-1 куст картошки. Самый быстрый картофель вырастает за 40 дней. Допустим экипаж 3 человека. Для половины потребностей требуется 15м2х05м земли + оборудование. Завозить такое количество на орбиту сразу не получится. В то же время удобрениями просто так почву не обновить. Я не говорю о том что в почве не то что много микроорганизмов, их дохуя, их не вынут, а некоторые так нужны там. Напомнить про Мир?
Кстати максимум что вырастили на мкс это хуёвая редиска.
До Мира и МКС было дохуя станций, разной хуёвости, были и наработки. Послать растить картошку на луну, можно но в 1 конец.
Аноним 09/06/16 Чтв 23:22:20  299095
>>299093
>земли
И сразу нет.
Аноним 09/06/16 Чтв 23:34:08  299099
>>299095
Вот этого двачую.
Бесит, когда тупые дети пытаются рассуждать о том, о чем вообще нихуя не знают. Совсем. 0.
Даже торчки-гроверы лучше разбираются в предмете.
Аноним 09/06/16 Чтв 23:48:22  299100
>>299099
С третьего раза прочитал не как "толчки", с пятого понял, что речь не о шайбе.
Аноним 10/06/16 Птн 00:50:12  299130
До 50-х годов про Марс лучше не заикаться.
Аноним 10/06/16 Птн 00:55:35  299134
>>299130
Ну блиа. Я хочу все-таки застать при жизни это событие
Аноним 10/06/16 Птн 00:56:50  299137
>>299134
Бросай курить.
Аноним 10/06/16 Птн 03:01:16  299154
Правда что в невесомости охуенная эрекция почти постоянно?
Аноним 10/06/16 Птн 03:09:30  299156
(155Кб, 356x500)
>>299154
Только когда Серовой нет рядом.
Аноним 10/06/16 Птн 03:19:44  299157
>>299156
Так ты эрегируешь от Марка? Неразделенная гомолюбовь не повод хейтить жену твоего ЕОК.
Аноним 10/06/16 Птн 03:35:00  299160
>>299157
Она же уродливая бабища. Фу.
Аноним 10/06/16 Птн 04:42:06  299164
Может ли пространство-время быть зацикленным?
Аноним 10/06/16 Птн 07:10:29  299168
Вопрос к умным анонам. Позавчера и вчера возвращался домой поздно вечером в Красноярске. Шёл примерно на запад, и наблюдал на небе перед собой слева "звезду" красноватого цвета, а справа - более холодного. Оба небесных тела довольно яркие (ярче звёзд). Что это за объекты? Красноватый - Марс? Не и вообще, как научиться такую инфу самостоятельно находить?
Аноним 10/06/16 Птн 09:08:21  299172
>>299087
в Дойчланде где-то, где Буран стоит
Аноним 10/06/16 Птн 09:08:47  299173
>>299164
не просто может, а есть
Аноним 10/06/16 Птн 09:09:13  299174
>>299168
Стеллариум установи на телефон и обдрачивайся
Аноним 10/06/16 Птн 09:11:25  299175
>>299168
Марс и Юпитер.
>как научиться такую инфу самостоятельно находить?
http://www.stellarium.org/ru/
http://homes.relex.ru/~zalex/main1251.htm
http://www.astrosurf.com/c2a/english/index.htm
Аноним 10/06/16 Птн 10:02:11  299177
>>299134
Анон, ты можешь хоть завтра погибнуть от кирпича на голову.
Аноним 10/06/16 Птн 10:50:34  299179
>>299174
>>299175
Спасибо, друзья.
Аноним 10/06/16 Птн 10:52:22  299180
>>299134
Изучай реальность и сможешь сам слетать хоть на Толиман.
Аноним 10/06/16 Птн 11:08:36  299182
>>299180
>Изучай реальность
>Толиман
Ты о чём?
Аноним 10/06/16 Птн 14:27:16  299189
>>299160
Вместо того что бы как тупая сучка поливать говном конкурентку, просто запишись в отряд испытателей и докажи Марку что ты лучше :3
Аноним 10/06/16 Птн 22:09:24  299215
>>298225 (OP)
Как называются оптические эффекты на фотографиях звезд типа полос по горизонтали и вертикали, всяких кружочков при взгляде сбоку и прочего?
Аноним 10/06/16 Птн 22:56:28  299222
>>299215
Блики?
Аноним 10/06/16 Птн 23:00:38  299224
>>299215
Приложи пример на фотках.
Аноним 11/06/16 Суб 03:06:07  299228
>>299215
Лэнс флейр.
Блик линзы.
Аноним 11/06/16 Суб 15:20:21  299232
(276Кб, 1024x1024)
>>299222
>>299224
>>299228
Именно это, спасибо. Т.е. получается, что объекты искажаются из-за зеркал или линз? В реальности же нет никаких лучей, отходящих от звезд. Нормально звезды выходят если только напрямую на матрицу фоткать?
Аноним 11/06/16 Суб 15:24:38  299233
>>299232
Это из-за подвесов, держащих линзу в йоба-телескопах.
Аноним 11/06/16 Суб 15:29:39  299234
>>299233
А ещё это может быть астигматизм.
Аноним 11/06/16 Суб 15:32:36  299236
>>299233
https://ru.wikipedia.org/wiki/%D0%94%D0%B8%D1%84%D1%80%D0%B0%D0%BA%D1%86%D0%B8%D0%BE%D0%BD%D0%BD%D1%8B%D0%B5_%D0%BB%D1%83%D1%87%D0%B8
Ага, это. На хаббле выходит та же херня?
Аноним 11/06/16 Суб 15:53:19  299243
>>299233
Не линзу, а вторичное зеркало. И ни на йоба, а на всех, где используется вторичное зеркало.
Аноним 11/06/16 Суб 15:56:20  299244
Это называется "дифракционный крест".
Аноним 11/06/16 Суб 16:00:18  299246
>>299234
Астигматизм так не выглядит.
Аноним 11/06/16 Суб 16:05:49  299248
(146Кб, 721x683)
>>298225 (OP)
Посоны, есть какой-нибудь сайт/симулятор, где можно было бы посмотреть как небо Земляшки выглядело в прошлом? А в очень далёком прошлом? А во времена динозавров? Ну и так далее.
Чтобы можно было понять какие звёзды были ближайшими, ярчайшими, как они взаимно располагались.
Аноним 11/06/16 Суб 16:21:05  299249
Я правильно понял что разница между соплом лаваля и клиновоздушным двигателем только в том, что лаваля можно сделать йобким только для какой то одной определенной высота или ебаться с навешиванием на сопло всяких движущихся насадок, а клиновоздушный йобкий на любых высотах и условиях, но не настолько йобок как лаваль сделанный под данную высоту или есть еще разница?
Аноним 11/06/16 Суб 16:23:30  299250
>>299249
О, и про люминь в качестве топлива поясните.
Вики пишет что он охуеть какое горючее, но из за того что он до какой то там температуры в 3К градусов остается твердым веществом он не совершает термодинамическую работа при расширении. Как такое может быть? Он же там в виде газа считай, а он по термодинамика адиабатой ебашить должен.
Аноним 11/06/16 Суб 16:29:34  299251
>>299248
Нет. Научных работ по этой теме мало. Это делается при помощи обратной n-body симуляции, а это нихуя не тривиальная штука, которая может упереться в точку неопределенности и дальнейшая "прокрутка назад" станет невозможной. Алсо, миллионы лет это весьма дохуя для сколько-нибудь точной симуляции. Сейчас вообще Меркурий считают способным сойти с орбиты через несколько млн лет с некоторой вероятностью. Более точных предсказаний тупо нет, обратных тем более. Несколько десятков-сотен тысяч лет еще имеются, и то узкоспециализированные, для звезд в ближайшем окружении.
Аноним 11/06/16 Суб 16:50:23  299254
>>299249
Все верно.

>>299250
>до какой то там температуры в 3К градусов остается твердым веществом
Штоа? У него температура плавления что-то около 600 с хуем, люминёвые провода даже в костре оплывают. Ты наверно с оксидом люминя попутал, который быстро нагорает пленкой на частицы (поэтому провода оплывают в мешочке из окислов и висят в пленке как гондон). Вообще, в APCP по процентному отношению люминя очемалая доля (~16% вроде, если не ошибаюсь), основная масса это окислитель. Поэтому да, есть такая проблема, но частицы мелкие и алюминия мало, а температура сгорания любой APCP-смеси больше 3К, так что вся хуйня успевает произойти. Потери на нерасширяющийся оксид алюминия присутствуют, в любом случае.

Кстати, лет 5 назад насо испытывало алюмо-водяную смесь ALICE. Не знаю до какой стадии оно дошло, но отзывались благоприятно, и таких проблем как APCP оно не испытывало. Смесь из разряда проще не бывает - замороженная суспензия очень тонких частиц алюминия в воде. Там алюминий соединяется с кислородом, а водород работает в качестве рабочего тела.
Аноним 11/06/16 Суб 16:58:14  299255
>>299249
Клиновоздушник немного труднее охлаждать и/или труднее облегчить.
Аноним 11/06/16 Суб 16:59:39  299256
Не могу посмотреть эту картинку http://images.ipac.caltech.edu/stsci/stsci-prc-2007-16-a/stsci_stsci-prc-2007-16-a_29566.jpg видимо из-за большого веса, просто крашит браузер. Если кто-то скачает то залейте на обменник.
Аноним 11/06/16 Суб 17:00:40  299257
(31Кб, 500x374)
>>299256
Правой кнопкой - сохранить.
Аноним 11/06/16 Суб 17:04:26  299258
(16Кб, 637x284)
>>299257
А теперь сам так сделать попробуй.
Аноним 11/06/16 Суб 17:05:52  299259
>>299258
Дебил правой кнопкой на ссылку, не открывая, в меню выбери "сохранить ссылку как"
Аноним 11/06/16 Суб 17:08:00  299260
>>299258
> хром вместо браузера
Ну дык что ты хочешь. У меня оно и так открывает нормально. Но вот на рыгхосте.
http://rgho.st/7mvqdKgc6
Аноним 11/06/16 Суб 17:44:13  299261
>>299251
Спасибо!
>Несколько десятков-сотен тысяч лет еще имеются, и то узкоспециализированные, для звезд в ближайшем окружении.
А где можно почитать или увидеть?
Аноним 11/06/16 Суб 19:19:50  299266
>>299261
Надо луркать по топовым тематическим журналам вроде ApJ, MNRAS, Icarus и т.п. Сходу не скажу, надо отсеивать arxiv и журналы на тему обратного моделирования и движения звезд и кластеров в рукавах - работ на эту тему много, но большинство исследует либо гипотетические варианты либо будущее, обратным моделированием мало кто занимается. Ну и про гипотетический пролёт звезды Шольца ты наверняка слышал. https://arxiv.org/abs/1502.04655
Аноним 11/06/16 Суб 20:11:48  299272
>>299254
https://ru.wikipedia.org/wiki/%D0%A2%D0%B2%D1%91%D1%80%D0%B4%D0%BE%D0%B5_%D1%80%D0%B0%D0%BA%D0%B5%D1%82%D0%BD%D0%BE%D0%B5_%D1%82%D0%BE%D0%BF%D0%BB%D0%B8%D0%B2%D0%BE
>В современных твердотопливных двигателях большой мощности чаще всего применяют смесь перхлората аммония с алюминием и каучуками. Иногда вместо каучуков используют полиуретан, что позволяет повысить срок годности шашки ТРТ и увеличить её жесткость, но, в ущерб технологичности производства. Алюминий является основным источником тепловой энергии благодаря высокой теплотворности реакции окисления. Однако ввиду высокой температуры кипения оксид алюминия в реактивной струе РДТТ является твердым веществом и не совершает термодинамической работы при расширении в сопле. Поэтому основным источником газообразных продуктов является полимерное связующее. Примесь твердых продуктов сгорания ТРТ увеличивает внутреннее трение в реактивной струе газов, что снижает КПД работы РДТТ. Удельный импульс такого топлива около 250-280 секунд.

>>299255
Не понял, почему? И его же проще собрать чем лаваля, да?
Аноним 11/06/16 Суб 20:42:37  299276
>>299272
>оксид алюминия
Я ж говорил что ты алюминь с оксидом попутал
Аноним 11/06/16 Суб 23:06:19  299290
У меня есть тупой вопрос, но не по теме космоса, а где спросить я не знаю. Можно спрошу у вас? Вы тут специалисты.
Аноним 11/06/16 Суб 23:16:41  299291
>>299290
Ну давай, хуле тут уже
Аноним 12/06/16 Вск 01:01:26  299294
В чём профит РК Зенит?
Читал что им выводили Фобос-грунт. Что пошло не так?
Аноним 12/06/16 Вск 01:31:52  299295
Поясните за CST-100.
Четыре жирных двигателя юзаются только как САС? Но зачем их тогда тащить на орбиту, неужели масса отстреливаемой конструкции с ними была бы такой жирной, что это нивелировало бы профиты?
Аноним 12/06/16 Вск 02:19:49  299299
Есть ли в космосе верх-низ? Ну то есть можно ли сказать, что одна звезда находится выше другой или что-то подобное.
Аноним 12/06/16 Вск 02:34:41  299300
>>299299
Когда мне было лет 5, я часто представлял, что если выйти за пределы земли можно провалится в низ в общую землю. Жудкое место тёмно-фиолетового цвета с непонятного вида сталогмиты.
Стоит ли говорить, что даже тогда я понимал, что это бред.
Аноним 12/06/16 Вск 02:56:34  299301
>>299299
нет.
Аноним 12/06/16 Вск 04:14:13  299308
>>299300
>я понимал, что это бред
Наоборот, это прозрение было. Гугли "инфрафизический слой" - да и в целом "Розу мира" читнуть не помешает.
Аноним 12/06/16 Вск 07:38:30  299309
как думаете MarsOne наебалово или реально полетят?
Аноним 12/06/16 Вск 08:05:40  299310
>>299291
В общем, один молодой человек постоянно говорит что через год половину европы и россии затопит т.к. таят ледники и еще что-то там. Это правда или очередной конец света?
Я это не слушал, а тут недавно на гиктайм прочитал что ледяные шапки почти растяли и температура растёт по часам.
Поясните, знатоки.
Аноним 12/06/16 Вск 08:29:48  299312
>>299309
Ну, да Винчи тоже нарисовал вертолет, а полетели сотни лет спустя.
Аноним 12/06/16 Вск 09:44:12  299314
>>298225 (OP)
Почему самым лучшим рабочим телом для ракеты считается водород, а не уран какой то? Уран же будет иметь намного больший импульс чем водород, при равной скорости.
Аноним 12/06/16 Вск 09:58:01  299317
>>299310
Неправда. Тает лед в воде, уровень моря не поднимается от этого (от таяния Гренландии и Антарктиды поднимается, но медленно, серьезные последствия будут лет через 30-50 - типа 200 миллионов беженцев из Бангладеш).
Аноним 12/06/16 Вск 09:59:05  299318
>>299314
Схуяли? Меньше атомная масса, больше уи, хуевее тяга.
Аноним 12/06/16 Вск 10:03:05  299320
>>299318
Поясни подробнее.
Аноним 12/06/16 Вск 10:31:12  299323
>>299317
Т.е. ближайшие лет 30 не стоит беспокоится?
Спасибо.
Аноним 12/06/16 Вск 11:05:52  299324
>>299309
>как думаете MarsOne наебалово
Это маняпроект, который с треском провалился. Они мечтали запустить реалити-шоу, которое будут смотреть по всему миру и бабло само потечет рекой. На это бабло они запилят транспорт и хижины и доставят все это на Марс. В итоге всем оказалось похуй и теперь они продают кружки и майки с надписью MarsOne. Короче, вместо медиа корпорации за миллиард долларов у них есть палатка на рынке в Мумбаи.
Аноним 12/06/16 Вск 11:17:02  299326
В сци сидят какие то дегенераты, поэтому спрошу тут.
Как из постоянного тока низкого напряжения сделать постоянный ток большого напряжения? Трансформатор ведь тут не годится, он на смене индукции переменного тока работает.
Аноним 12/06/16 Вск 11:26:26  299328
>>299326
http://www.chipdip.ru/video/id000323327/
Аноним 12/06/16 Вск 13:47:42  299336
(16Кб, 709x287)
>>299326
Универсальный рецепт: делаешь генератор, далее повышающий трансформатор, далее выпрямитель.

Специализированный рецепт: генератор, умножитель напряжения как на пикче. У него будет слабее ток отдачи чем в первом случае, зато высокие напруги получить проще.

Забавный рецепт: берёшь хуеву тонну оптопар светодиод-фотодиод, соединяешь светодиоды параллельно, фотодиоды последовательно.

Из той же оперы: заряжаешь хуеву тонну кондеров от низкого напряжения, соединяешь последовательно. В целом умножительно напряжения на пикче это что-то вроде автоматизированной реализации этого принципа.
Аноним 12/06/16 Вск 13:49:46  299337
>>299324
Дурачки блин. Это ж так дохуя просмотров было бы в эпоху интернета. Та даже ещё один пилотируемый на Луну.
Аноним 12/06/16 Вск 14:48:40  299348
>>299337
Осталось решить некоторые мелкие проблемы. Например найти пару сотен миллиардов зелени. Но это же сущие пустяки.
Аноним 12/06/16 Вск 15:31:45  299355
>>299348
Не умеют ребята развести инвестора. Или бизнес-план говно. Вдруг они там решили кружки продавать даже получив миллиарды. Вроде бы денег им даже кто-то дал, но они спустили их без пользы. Оно в принципе ясно, задача новая, нетривиальная. Видимо как с искусственным интеллектом будет, волнами накатывать инвесторский/журналистский интерес.
Аноним 12/06/16 Вск 15:58:51  299360
>>299324
Один хуй, видимо чуваки что-то да заработали.
Пидоры, в общем. С самого начала считал это всё наебаловом, с такими-то сроками и бизнес-планом.
Аноним 12/06/16 Вск 19:12:30  299469
>>299355
Проблема в том что таких инвесторов в принципе нет
Аноним 12/06/16 Вск 19:17:47  299473
Есть ли смысл ехать в лес в отдалении около 5км от города(да, мухосрань) для наблюдений? Есть ли разница(вроде зона зеленая, но город близко же).
Аноним 12/06/16 Вск 19:20:40  299477
(5306Кб, 900x692, 00:00:36)
Задавайте ответы.
Аноним 12/06/16 Вск 19:26:04  299485
>>299473
В сторону города наблюдать смысла не будет, в противоположную будет.
Аноним 12/06/16 Вск 20:55:35  299548
>>299477
Что за фильм?
Аноним 12/06/16 Вск 21:35:22  299573
>>299310
Если растаят все ледники Антарктики и Арктики уровень Мирового океана поднимется примерно на 60 метров. Пострадают страны, как ты понимаешь, расположенные на высоте ниже этого уровня. Пизда Нидерландам, Дании, большой части Японии. Африка, как самый "высокий" материк, почти не изменит своих очертаний. Рашка тоже пострадает незначительно - большой кусок наступающий океан отожмёт у Западной Сибири. Но вся эта хуйня случится, если случится (на самом деле ничего тут невероятного нет - на Земле чередуются межледниковые периоды и периоды оледенения. Были времена, когда Земля была полностью свободна от ледяных шапок - например в меловой период. А были, когда почти полностью покрывалась льдом) в сроки сопоставимые со временем существования самой человеческой цивилизации. То есть несколько тысяч лет.
Вот здесь можешь посмотреть высоту над уровнем моря своей Мухосрани
http://elevationmap.net/
А здесь гипотетические очертания материков в случае, если все ледяные шапки Земли растаят
http://ngm.nationalgeographic.com/2013/09/rising-seas/if-ice-melted-map
Аноним 12/06/16 Вск 22:02:16  299585
Платина: что почитать для полноценного вкатывания в тематику? Помимо Кононовича и Морозова
Аноним 12/06/16 Вск 22:10:43  299595
Хочу американскую жену из NASA, только не биолога какого-то.
Двач, помоги. Где знакомиться, какие подводные камни, как показать, что ей нужен именно няшный кун из восточной Европы, а не ерохин из Алабамы?
Аноним 12/06/16 Вск 22:19:57  299598
>>299314
Чем меньше молекулярная масса, тем выше скорость истечения, т.к. придать высокую скорость легкой молекуле проще чем тяжелой, при одинаковой энергетике реакции. А именно скорость истечения определяет эффективность двигателя, она эквивалентна удельному импульсу.
Аноним 12/06/16 Вск 22:26:32  299602
Если планеты кружатся вокруг планет, планеты вокруг звёзд, звёзды вокруг цента галактики, то вокруг чего кружатся галактики?
Аноним 12/06/16 Вск 22:32:12  299605
>>299469
Ну вот тут хуй знает. Не берусь утверждать того чего не знаю. Инвестор, как говорится, не мамонт.
Аноним 12/06/16 Вск 22:33:11  299606
>>299602
Вокруг центра масс группы галактик. (Это небольшое скопление ближайших соседних галактик)
Группы галактик вокруг центра масс скопления галактик.
Скопления галактик вокруг центра масс сверхскоплений галактик.
Аноним 12/06/16 Вск 22:34:20  299607
>>299602
Вообще говоря всё движется в эдаком гравитационном супе, даже твоя собака ты же не пидар какой-нибудь, верно?. И центры масс это такой способ упрощать рассчёты.
Аноним 12/06/16 Вск 22:39:29  299608
>>299602
> планеты кружатся вокруг планет
Планетоиды кружатся, согласно определению IAU.
>звёзды вокруг цента галактики
Звезды в галактике кружатся вокруг друг друга, точнее общего барицентра. Центр галактики как скопление говн и иногда черной дыры им похуй, т.к их масса обычно ничтожна по сравнению с общей массой звезд галактики.
>вокруг чего кружатся галактики?
Вокруг друг друга, опять же.

На самом деле никто не кружится вокруг чего-либо, а просто движется по инерции, отклоняясь гравитационным градиентом. Например, звезды в плотных скоплениях могут двигаться по совершенно хаотическим (со стороны) траекториям. Как >>299607 сказал, все плавает в гравитационном супе.А учитывая, что гравитация распространяется с черепашьей для суперструктур скоростью света, конфигурация этого супа может быть весьма причудлива, и в гравитационных волнах может быть запасено дохуя энергии, которая может делать многие локальные системы незамкнутыми.
Аноним 12/06/16 Вск 23:00:13  299615
>>299608
>Звезды в галактике кружатся вокруг друг друга, точнее общего барицентра. Центр галактики как скопление говн и иногда черной дыры им похуй, т.к их масса обычно ничтожна по сравнению с общей массой звезд галактики.
Тут пока не совсем ясно, что к чему. Звезды в спиральных галактиках вращаются как прибитые к спицам, с линейной скоростью, мало зависящей от расстояния до центра галактики. Наблюдения Цвикки (и сто тысяч более поздних), парадокс вириала, все другие вещи, необъяснимые излучающей барионной материей, - все они заставляют предполагать, что галактики шарообразны, основная масса содержится в их гало (в виде темной материи), и вообще, творится абсурд и коррупция.
Аноним 12/06/16 Вск 23:06:40  299618
>>299615
>что галактики шарообразны, основная масса содержится в их гало
Тем не менее звезды в спиральных галактиках образуют диск. Шариками обычно эллиптические галактики.
Аноним 12/06/16 Вск 23:21:09  299621
>>299618
>Тем не менее звезды в спиральных галактиках образуют диск
Звезды - да. Но они излучают, и их можно наблюдать. Определение астрономии жи. Но вот их движение пока можно объяснить лишь присутствием на порядок превосходящей несветящейся массы в гало.
Аноним 12/06/16 Вск 23:25:04  299623
>>299621
Ну а вопрос-то теперь прочитай. >>299602
Гало - хуело, темная материя - темная хуерия, зачем ты отвечаешь на вопрос, который вообще не задавали?
Аноним 12/06/16 Вск 23:41:40  299624
>>299623
>Гало - хуело, темная материя - темная хуерия, зачем ты отвечаешь на вопрос, который вообще не задавали?
Я отвечал на посты (типа этого - >>299608), а не на затерявшийся вопрос.

Ну и не забудь про "призрачную энергию": 75% массы Метагалактики приходится на нее. Вокруг чего же такого массивного вращаются галактики?
Аноним 12/06/16 Вск 23:47:11  299627
>>299624
Вокруг общего центра масс.
Точка. Из чего они там блядь состоят - дело десятое.
Аноним 12/06/16 Вск 23:50:25  299628
>>299627
>Вокруг общего центра масс.
>Точка. Из чего они там блядь состоят - дело десятое.
Это очень радует, что ты так свободно обходишься с фактором, увеличивающим постоянную Хаббла. Ну ок, масса так масса.
Аноним 12/06/16 Вск 23:55:25  299630
>>299602
Вокруг твоей мамки очевидно же.
Аноним 13/06/16 Пнд 00:12:40  299634
>>299628
У тебя темная энергия имеет массу, тебе бы пора таблетки принять, или съебать отсюда в ужасе уже.
Аноним 13/06/16 Пнд 00:21:01  299636
Так всё-таки с какой скоростью распространяются гравитационные волны?
Аноним 13/06/16 Пнд 00:21:59  299637
>>299636
Мгновенно.
Аноним 13/06/16 Пнд 00:23:57  299640
>>299636
Небольшие возмущения со скоростью света. Значительные возмущения уже сами искажают пространство и там не так просто, но можно считать что скорость света тоже примерно.
Аноним 13/06/16 Пнд 00:32:51  299642
>>299634
>У тебя темная энергия имеет массу, тебе бы пора таблетки принять, или съебать отсюда в ужасе уже.

>Согласно опубликованным в марте 2013 года данным наблюдений космической обсерватории «Планк», общая масса-энергия наблюдаемой Вселенной на 95,1 % состоит из тёмной энергии (68,3 %) и тёмной материи (26,8 %)[3][4][5].
Аноним 13/06/16 Пнд 01:37:01  299658
Значит ли что-нибудь, то что ТМ ровно в 5,5 раз массивнее обычной материи?
Аноним 13/06/16 Пнд 02:04:42  299661
Если глаз передаст мозгу информацию о, ~четверть секундное пятно света, Будет ли это означать нечто космическое или это всего лишь внутренние механизмы глаза?
Аноним 13/06/16 Пнд 02:11:46  299663
Имеется ввиду внутри здания, на против уличной стены и зашторенным окном.
>>299661
Аноним 13/06/16 Пнд 02:34:42  299666
>>299640
В общем можно точно сказать что в любом случае не быстрее скорости света?
Аноним 13/06/16 Пнд 02:45:44  299667
>>299661
Может быть как космическое, так и твои фантазии. Даже самые высокоэнергетичные йобы не больше жирной точки. Пятно куда больше смахивает на фантазии.
Аноним 13/06/16 Пнд 02:55:30  299668
>>299667
Т.е. в редком случае мелкая точка на глазу это ёба, а не фантазии?
Аноним 13/06/16 Пнд 03:33:29  299673
Бывают ли командиры космических станций из гражданских?
Аноним 13/06/16 Пнд 05:23:45  299681
>>299673
Зачем? В смысле это опасно, ввиду их якобы ненадёжности. Утверждать не стану, но у военного куда больше доверия, хотя бы статистически.
Аноним 13/06/16 Пнд 13:29:39  299699
(1238Кб, 500x494)
Свет не может вырваться из черной дыры потому что там сила притяжения равна скорости света. Возможно ли, что свет не просто не может вырваться, но и постепенно всасывается в ЧД из-за того, что фотоны имеют массу?
Да-да, я не ошибся, фотон имеет массу
https://ru.wikipedia.org/wiki/%D0%A4%D0%BE%D1%82%D0%BE%D0%BD
Аноним 13/06/16 Пнд 13:52:42  299700
>>299699
Фотон имеет массу только во время своего движения. Масса "стоячего" фотона равна нулю.
Аноним 13/06/16 Пнд 13:56:53  299701
>>299700
В черной дыре фотон не перестает двигаться (без движения фотон вообще не существует, если уж на то пошло), просто гравитация мешает ему продвинуться дальше.
Аноним 13/06/16 Пнд 14:14:04  299703
>>299701
Тогда у меня вопрос, ты сказал, что "свет постепенно всасывается в ЧД из-за того, что фотоны имеют массу". Да, Чёрная дыра поглощает свет. Да, фотоны имеют массу ( исключительно в движении, как мы уже выяснили). Но где связь между этими двумя фактами?
Аноним 13/06/16 Пнд 14:49:49  299710
Получается что фотон существует когда движется, а когда нет исчезает., когда же он приближается к ЧД он начнёт нарезать круги и настанет такой момент, когда луч будет перпендикулярен силе тяжести и замедляясь луч пересекает скорость =0 тем самым пропадая. Возможен ли такой расклад?
Аноним 13/06/16 Пнд 14:58:38  299711
>>299703
Мне вот что интересно: сила притяжения у черной дыры равна скорости фотонов с массой или без массы? И вообще, в формулах по физике учитывают, что фотоны имеют массу?
Аноним 13/06/16 Пнд 14:59:28  299713
>>299668
Да.
Аноним 13/06/16 Пнд 15:50:48  299715
Я так понял, что нейтрино детектируют оптической камерой, по взаимодействии со льдом. Так в чём сложность сделать кучу детекторов подобного рода? И на что помимо льда нейтрино реагируют.
Аноним 13/06/16 Пнд 15:56:07  299716
>>299710
Хватит бредить, пойди на белый карлик пробздись штоле.
Аноним 13/06/16 Пнд 16:11:37  299719
>>299681
Т.е. космонавтом тру только военный стать может?
Аноним 13/06/16 Пнд 16:33:03  299723
>>299719
По сути не космонавтом, а пилотом КА. Комонавт это любой пассажир, побывавший на орбите .
Опять же моё предположение.
Аноним 13/06/16 Пнд 16:37:45  299725
>>299711
Масса не важна. Чёрная дыра не даст улететь ни чему, что медленнее или равно скорости света. Скорость, важна только скорость. Это уже прикол нашей физики, что только безмассовая частица может в эту скорость.
Аноним 13/06/16 Пнд 16:39:29  299726
>>299715
Детекторов и делают до хрена. Миллионы тонн чистейшего дистиллята. А материал должен быть плотен, прозрачен, в огромных количествах, а потому и дешев. Потому и используют воду. Стекла и прочее это охренеть как дорого без всяких преимуществ.
Аноним 13/06/16 Пнд 16:39:30  299727
Господа, можете посоветовать книг каких нибудь по астрономии?
Аноним 13/06/16 Пнд 16:40:44  299728
>>299727
>Господа, можете посоветовать книг каких нибудь по астрономии?
Да, учебник.
Аноним 13/06/16 Пнд 16:46:49  299729
>>299728
Можешь посоветовать какой то конкретный?
Аноним 13/06/16 Пнд 16:47:45  299730
>>299729
За 9 класс.
Аноним 13/06/16 Пнд 16:47:52  299731
>>299723
Только что нагулил, что в совке еще в 60-х был первый гражданский командир.

ЫЫЫЫЫЫЫЫ
Аноним 13/06/16 Пнд 16:48:30  299732
>>299729
>Можешь посоветовать какой то конкретный?
В твоем случае - без разницы. Что-нибудь для средней школы подойдет.
Аноним 13/06/16 Пнд 17:59:03  299743
(119Кб, 1280x720)
Спейсаны, на картинке на заднем плане между героями виден нахлобученный на белую ракету крылатый апарат. Это что-то имевшее реальный прототип (хотя бы на уровне влажных фантазий типа Клипера или Гермеса), или это только воображение художника?
Аноним 13/06/16 Пнд 18:31:00  299747
>>299743
Не имело, т.к. нет хвоста. Но вообще по форме это нечто среднее между X-20, X-37 и RLV. Во всяком случае эта линия, т.к. не похоже ни на клипер/ixv, ни на спираль/дримчейзер. Ни даже на HOPE/HOPE-X. Может среди каких-то совсем уж маняконцептов и отыщется нужный, но сомневаюсь что они руководствовались каким-то реальным прототипом, просто нарисовали похоже и всё.
Аноним 13/06/16 Пнд 18:32:10  299748
>>299747
Хотя вместо хвоста вертикальное оперение может быть на концах крыла, с такого ракурса не совсем ясно. Тогда ближе всего к X-20.
Аноним 13/06/16 Пнд 19:31:00  299753
Поясните вот что: от чего зависит скорость частиц, таких как свет или звук?
Почему у света такая высокая скорость, а у звука не очень?
А нельзя частичу света замедлить? Или она всегдаидёт в 1 направлении со своей скоростью?
Аноним 13/06/16 Пнд 20:20:44  299761
>>299731
Зато он был патриотом!
Аноним 13/06/16 Пнд 20:50:58  299763
>>299761
Есть еще гражданские??

Кстати - а кто пизже на борту - бортинженер/командир или ученый? Командир/бортач может в науку? Или они онли по матчасти?
Аноним 13/06/16 Пнд 20:53:16  299765
Когда частники свои ракеты пускают - это надо согласовывать с военными же?

И вот еще интересно - допустим частная фирма (США) запилила орбитальную йобу для полета вокруг Земли. Но что то пошло не так и она вынуждена приземлиться в России/Китае. Ее на подлете военные не собьют? Как предупредить их?
Аноним 13/06/16 Пнд 21:24:21  299770
https://www.youtube.com/watch?v=IJhgZBn-LHg
Тут всаус по унылой вебмке целое видео запилил.
Вот.
Аноним 13/06/16 Пнд 21:45:09  299778
>>299765
1. Технически, все запуски объявляются в реестр UNOOSA, орган ООН, отвечающий за разруливание кто чего запустил на орбиту.
http://www.unoosa.org/oosa/en/spaceobjectregister/index.html
На практике из-за тамошней бюрократии стадия обработки заявки может длиться пару месяцев, даже когда аппарат уже давно крутится. UNOOSA работает на уровне наций, поэтому самому туда не получится подать документ. Только представитель соответствующей страны-участника ООН может сделать это. Иногда за владельца спутника подают инфу запускающие, так что реестр этот штука довольно приблизительная. Заявленные орбиты там тоже никем не проверяются, в частности вояки (в основном американские) часто шлют дезу про свои военные спутники.

2. На практике все частники получают сертификат от регулятора, что ракета может летать и к полётам допущена, как и самолёты. Каждый запуск РКН согласуется с регулирующим органом. Естественно, что военные стран, у которых есть СПРН, участвуют в соответствующем органе-регуляторе, плюс военные разных стран обмениваются инфой о пусках, чтобы не приняли за ядрён батон или еще чего нехорошее. В штатах с середины 80-х, когда разрешили частникам пускать, есть упрощенная процедура - прошедшие сертификацию (должны иметь ЦУП, FTS и т.д.) могут только информировать о пуске. Публикуются соответствующие NOTAM'ы для самолётов.

3. Основные страны ведут все более-менее крупные объекты на орбите, плюс есть еще независимые сети наблюдения вроде ПулКОНа или SST, и отдельные инструменты. Данные СККП в основном засекречены, в гражданский доступ попадают исключения, единого каталога нет. NORAD/USSN публикует гражданскую версию своего реестра на spacetrack.org. Там искажены или отсутствуют орбиты секретных спутников, да и формат TLE не очень точен.

В общем неизвестному объекту сойти с орбиты невозможно - все это отслеживается от запуска до схода, так что предупреждать никого не приходится.
>Ее на подлете военные не собьют?
Было б чем, лил.
Аноним 13/06/16 Пнд 21:45:21  299779
>>299770
А нет, я соврал. Там нет ничего такого.
Аноним 13/06/16 Пнд 22:10:19  299786
>>299753
Звук не частица.
Фотон не имеет массы.
Можно. Гугли скорость сета в веществе.
Аноним 13/06/16 Пнд 22:15:14  299788
>>299763
В науку могут все. Только ради неё люди ещё в космос и летают. На борту субординация невиданная на Земле. Есть зоны ответственности, и в них капитан вне ЧП никогда не вмешается, все уже взрослые, хотя последнее слово всегда за ним.
Аноним 13/06/16 Пнд 22:27:56  299790
(8493Кб, 640x360, 00:01:03)
>>299778
Ну когда это йоба будет так на высоте км 30-40

>>299788
Короче - я авизадрот, но и космические полеты интересуют тоже. Учился дома в шараге, перевелся в гейропу на последний курс физики, и конечно же не потянул лол. Я вообще собирался тут один год, а потом на инженера попиздовать . Но получилось так, что 2 года проебал тут, могут пидорнуть. Но я не сдаюсь, и поеду доки в школу инженерную подавать.
Мне вот интересно, ради лулзов только - если я тут вдруг выучусь таки, останусь, насобираю на пилотское и т.п. В гейропе часто отбор астронавтов идет или нет? С пломбами берут?
Аноним 13/06/16 Пнд 22:34:51  299791
>>299790
Там даже в очках берут.
Аноним 13/06/16 Пнд 22:52:14  299792
>>299790
USA! USA!
Сириусли, подумай.
А вообще, шансов больше было в РФ, лол. Они проводили первый открытый набор недавно, и пришлось продлевавать несколько раз, ибо кандидатов мало было. Говорят, теперь снова всё закрытым будет.
Аноним 13/06/16 Пнд 22:55:48  299793
>>299791
Ну зрение то хорошее может и не нужно то на корабле, а вот про пломбы хз.

>>299792
Да мне еще тут года 4 учиться лол.
Аноним 14/06/16 Втр 00:15:27  299796
Читаю тут про магнитное пересоединение и периодически вижу в комментариях реплики, мол, Солнце ни разу не ядерное, а вообще электрическое. Может кто пояснить за эту теорию?
Аноним 14/06/16 Втр 00:20:33  299797
(60Кб, 525x328)
>>299796
Што?
Аноним 14/06/16 Втр 00:24:14  299798
>>299796
>Солнце ни разу не ядерное, а вообще электрическое. Может кто пояснить за эту теорию?
Могу пояснить, что всякое говно читать не надо, так же как мамки говорят своим личинусам, что не стоит все подряд тянуть в рот.
Аноним 14/06/16 Втр 00:59:43  299801
>>299699
Нет. Свет никак не притягивается чёрной дырой. Рядом с ней искажается пространство. Если ты имеешь в виду E = pc^2, то эта часть "массы" не является массой в обычном понимании, то есть не воздействует с гравитацией. Ставился опыт для этого, наблюдались искажения движения фотонов от фоновых звёзд при прохождении рядом с солнцем. Пути искажались, но все фотоны все цветов (=энергий) проходили одинаковые траектории.
Аноним 14/06/16 Втр 07:03:27  299817
>>299790
Пиздец, уехал в Европу, чтобы стать космонавтом, лол. Не удивительно, что ты в физику не смог, ты ж тупой нахуй!
Аноним 14/06/16 Втр 11:45:04  299828
>>299817
Щито? Я сюда просто перевелся, так получилось. А про космос для лулзов спросил, мало ли
Аноним 14/06/16 Втр 12:05:17  299832
>>299790
Что за песня на вебм?
inb4 darude - sandstorm
Аноним 14/06/16 Втр 12:56:17  299838
Почаны, тред не читал, лень гуглить спрошу у анона. Вот есть черные дыры. С маленькими все понятно, пересечь горизонт событий у них хуй получится, как, в принципе, и нормально приблизиться к нему - разорвет к хуям. А вот если взять сверхмассивные - допустим мы приближаемся к горизонту событий сверхмассивной ЧД. Для стороннего наблюдателя мы будем замедляться и краснеть, пока не застынем на горизонте, а для нас наоборот - время будет ускоряться, притом, нихуево так ускоряться. Значит ли это, что находясь на предельно малом расстоянии от горизонта событий, время ускорится настолько, что мы увидим всю историю Вселенной, вплоть до её смерти?
Аноним 14/06/16 Втр 13:24:00  299841
>>299801
>Если ты имеешь в виду E = pc^2, то эта часть "массы" не является массой в обычном понимании, то есть не воздействует с гравитацией.
Как такое возможно? Эта масса какая-то особенная?

>Пути искажались, но все фотоны все цветов (=энергий) проходили одинаковые траектории.
Не совсем понял. Что значит одинаковые траектории? Извините за тупость.
Аноним 14/06/16 Втр 16:05:32  299852
>>299841
По твоей логике гравитация тянет фотоны за массу. Соответственно чем больше "масса" тем сильнее должно притянуть. Дисперсия должна получиться. Но её нет. Совсем. У фотона нет массы. А то энергия, а не масса.
Аноним 14/06/16 Втр 16:47:03  299857
>>299852
>А то энергия, а не масса.
Но по знаменитой формуле Эйнштейна, если есть энергия, то должна быть и масса.
Аноним 14/06/16 Втр 17:16:15  299860
>>299857
Нет.
Аноним 14/06/16 Втр 17:24:12  299862
>>299860
Да
Аноним 14/06/16 Втр 17:25:11  299863
>>299852
И у фотона есть масса:
https://ru.wikipedia.org/wiki/%D0%A4%D0%BE%D1%82%D0%BE%D0%BD
Аноним 14/06/16 Втр 17:32:08  299864
Почему с доски выгнали тред по Kerbal Space Program, но при этом оставили Space Engineers? Второй же вообще никакого отношения к тематики доски не имеет. Совсем никакого.
Аноним 14/06/16 Втр 17:33:57  299865
>>299863
> Фотон — безмассовая нейтральная частица
Што
Аноним 14/06/16 Втр 17:36:22  299866
>>299863
Это масса покоя.
>>299865
То
Аноним 14/06/16 Втр 17:37:13  299867
(17Кб, 286x34)
>>299865
Аноним 14/06/16 Втр 17:38:56  299868
>>299866
>Это массаса покоя.
Наоборот, масса покоя фотона = 0, а при движении равна числу на скрине выше.
Аноним 14/06/16 Втр 17:43:41  299869
>>299868
>а при движении равна числу на скрине выше.
Ни одна частица обладающая массой не может достичь скорости света.
Фотон же наоборот, не способен двигаться со скоростью отличной от скорости света.
Аноним 14/06/16 Втр 17:44:55  299870
>>299869
>Ни одна частица обладающая массой покоя > 0 не может достичь скорости света.
Пофиксил.
Аноним 14/06/16 Втр 17:55:17  299871
>>299870
Ой всё
Аноним 14/06/16 Втр 20:58:09  299879
Черные дыры движутся относительно других черных дыр ?
Аноним 14/06/16 Втр 21:08:36  299884
Вот если мы приближаемся к скорости света, время вокруг нас течет все быстрее, на скорости 99,9999% за 1 мою секунду вокруг пройдет миллион лет во вселенной. Для стороннего наблюдателя я наоборот как бы застыну, но буду очень быстро нестись, не меняя своих физических значений.
------------------------------------
Значит если взять фотон, который движется со 100% скорости света, то он для нас как бы заморожен во времени (физическое состояние его не меняется) но очень быстро движется в пространстве.
А мы для фотона двигаемся бесконечно быстро, что он за мгновение видит всю историю вселенной, но как тогда объяснить его путешествие во вселенной с точки зрения наблюдения из глаз фотона ? Для него мы движемся бесконечно быстро или же все таки конечно быстро ?
Аноним 14/06/16 Втр 21:31:11  299891
>>299884
Мы для фотона даже не существовали никогда. Он за мгновение видит не всю историю вселенной, а всю историю которая когда-либо будет вообще до самой бесконечности. И от самого бесконечно большого отрицательного значения до создания мира.
То есть- 0. То что мир для фотона вообще не существует гораздо проще и логичней, и скорее всего верней.
Диванный кукаретик
Аноним 14/06/16 Втр 21:34:02  299893
>>299891
А откуда берутся фотоны?
Аноним 14/06/16 Втр 22:38:47  299903
>>299879
Да
Аноним 14/06/16 Втр 22:39:08  299904
>>299891
Иди нахуй отсюда.
Аноним 15/06/16 Срд 00:30:11  299931
>>299893
Из твоей мамки.
Аноним 15/06/16 Срд 01:26:19  299940
>>299904
Все он верно написал.
Аноним 15/06/16 Срд 04:02:20  299964
>>299941
Лучше тебе репорт кину.
Аноним 15/06/16 Срд 06:37:06  299966
1) Это тупых вопросов тред?
2) Кто отвечакт?
3) Перекат запилят?
Аноним 15/06/16 Срд 07:22:47  299967
>>299838
Космический бампец вопросу
Аноним 15/06/16 Срд 07:23:47  299968
>>299879
Ты охуеешь, но они еще и вращаются
Аноним 15/06/16 Срд 10:33:15  299972
>>299966
1. Да
2. Александр Друзь
3. Да
Аноним 15/06/16 Срд 10:38:43  299973
>>299972
Тогда вот еще вопрос:
В какой теории возможно управлять гравитацией? И как это будет происходить исходя из верности теории?
Аноним 15/06/16 Срд 10:42:02  299974
>>299838
И еще вдогонку вопрос, пока не перекатил тред никто. Вот есть нейтронные звезды. В них вещество настолько сильно корежит, что оно превратилось в одну большую кашу из нейтронов. И, если сила тяжести перевешивает силу газов изнутри звезды АЗАЗАЗ В ПИРДАКЕ У ТИБЯ ГАЗЫ)))000, то она может схлопнуться в черную дыру. Но, как мы знаем, нейтронные звезды еще и вращаются с неебитческой скоростью. Так вот в чем вопрос, может ли такое случиться, что звезда будет вращаться настолько быстро, что даже падение силы газов не повлечет превращение её в ЧД за счет просто охуительнейшей центробежной силы вращения? Что с ней тогда станет? Нейтроны на кварки распадутся?
Аноним 15/06/16 Срд 11:17:29  299977
Почему наша планета настолько петушиная что не имеет гор выходящих за пределы атмосферы?
Аноним 15/06/16 Срд 11:24:54  299979
>>299977
Высота гор определяется их материалом и силой тяжести на самой планете. Так вот предельная высота горы из гранита из чисто механических расчетов получается около 9000 метров для Земли. Надо ли говорить, что это отличное совпадение с реальностью?
Аноним 15/06/16 Срд 11:34:26  299980
>>299977
Атмосфера на высоте 100 км заканчивается. Кому нужны такие горы то?
Аноним 15/06/16 Срд 11:35:17  299981
>>299979
А почему так? Было бы круто иметь гору 100 км высотой
Аноним 15/06/16 Срд 11:35:47  299982
>>299980
Запускать ракеты.
Аноним 15/06/16 Срд 11:37:24  299984
>>299982
Планете плевать на цивилизацию.
Аноним 15/06/16 Срд 11:38:29  299985
>>299981
Тебе же сказали, что горы бОльшего размера будут либо разрушаться, либо просто проваливаться вниз.
Аноним 15/06/16 Срд 11:56:36  299986
>>299974
Гугли Блицар. Помимо этого, нейтронная звезда не факт что состоит из нейтронов. На самом деле никто не знает из чего она состоит.
Аноним 15/06/16 Срд 11:59:09  299987
А какой предел у железнорудых гор полностью состоящей из оной породы?
Аноним 15/06/16 Срд 11:59:50  299988
Вопрос про ремонт хаббла.
Шаттл туда долетел потому что почти пустой шёл или в него дополнительный топливный бак засунули?
Орбита Хаббла вроде высоковата.
Аноним 15/06/16 Срд 13:09:31  299998
>>299988
Мало того, они еще и бустанули орбиту телескопу.
>долетел потому что почти пустой шёл
Это.
>или в него дополнительный топливный бак засунули?
Неа. У него есть запас по дельте после вывода на орбиту, да и зачастую есть на чем сэкономить при рандеву.
Аноним 15/06/16 Срд 14:00:27  300005
Используются ли какие-то мистические практики в космической науке? Шаманство, магия, заговоры?
Аноним 15/06/16 Срд 14:17:09  300006
>>299974
Бтитард вывел существование блицаров на коленке из говна и палочек, лол. Двач - на острие науки.
Аноним 15/06/16 Срд 16:24:23  300014
(1949Кб, 390x248)
>>299973
Нет пока такой теории. Есть пока теории принципа действия гравитации. Их несколько и ни одна не подтверждена даже близко. Из наиболее известных и рассматриваемых официальной физикой присутствует петлевая квантовая и струнная. Петлевая квантовая пытается связать все взаимодействия в рамках стандартной модели, причем вариант петлевой пытается все четыре варианта фундаментальных представить как вариации чего-то одного. Мне лично стандартная модель больше импонирует, но кажется таким наиболее упрощенным выбором для консервативных старых пердунов. Я сам по натуре больно консервативен, вероятно поэтому стандартная модель так притягательна на мой взгляд, но кажется слишком простой, чтобы оказаться верной. Струнная же теория такая для угорающих по молодости и хардкору, как мне видится. Смелая и в какой-то мере даже наркоманская. Слишком "майнд-фри" конструкции в себе содержит. Многомерность сначала от 11 измерений потом аж до 26-ти, и видимо это только начало. Некоторые эксперименты на БАК опровергли несколько установок и следом опровергли некоторые теории в области струн, оттого фанатов у нее в последнее время поубавилось, но это вовсе не значит, что на ней можно ставить крест.

Суть в том, что стандартная модель слишком проста и хочет притянуть объяснение и получение сути всего мироустройства привычной методологией последовательного накопления знаний, коей человеки много веков вполне успешно пользовались. Струнная же теория предлагает новый подход миропознания, когда вся физическая теория строится чуть ли не исключительно на математической абстракции. Правда, как водится, где-то по середине, если вообще где-то водится. Мне вообще нотки агностицизма с возрастом стали близки. Но только я имею ввиду настоящий агностицизм. Не вот этот позерский "я не атеист и не верующий, я агностик"! А тот самый, который подразумевает, что предел познания существует, и что возможно человечеству самую подкорку мироустройства так и не удастся всковырнуть. И скорость света - настоящий и непреодолимый предел скорости для человека. И ни пузыри, ни прочие лучшие умы Альбукерке не помогут в этом.

А управление гравитацией, или т. н. "искусственная гравитация" - одно из основных запретных для твердого нф допущений. Их всего два по сути и второе собственно сверхсвет. Официальная наука искусственную гравитацию в разряде реализуемых на данный момент вариантов не рассматривает. Поэтому и нет никаких теорий на этот счет. Есть один известный способ управления гравитацией, и это управление движением масс. Собственно пройдя из своей сычевальни на толкан ты уже управляешь гравитацией, изменяя вектор силы тяжести Земли на долю в 10^"минус несколько десятков с копейками". Так что тут конкретизировать надо, что ты подразумеваешь. Создание гравитационных полей без соответствующих масс-энергий - такая же научная ересь как и эфир. Но тут важно отделять "журналистский" термин. Например видел в каком-то западном издании, как искусственной гравитацией называли собственно центробежную, создаваемую в гипотетических вращаемых КА. Это не гравитация, это центробежная сила, которая по сути не центробежная, потому что этот термин серьезными физиками также не принимается, но это уже совсем другая история.
Аноним 15/06/16 Срд 16:36:09  300016
Было два вопроса, но один забыл, пиздец, сидел в толкане, придумал вопрос, но сука ипхона под рукой не было, чтобы в заметки написать и все, пизда теперь вопросу. Забыл нахуй. Поэтому только один. Собственно Сатурн-5. Правильно ли я понимаю, что эта огромная йоба вывела на орбиту полезную нагрузку, а потом эта полезная нагрузка сама себя разогнала с орбиты до почти второй космической, потом затормозила с 11 км/с до приемлемых для лунной орбиты 1.6 км/с или около того. А потом взлетела до 2.4 км/с? Или больше. Как вообще лунный модуль на Землю возвращался? С около лунной второй космической или больше?
Аноним 15/06/16 Срд 16:58:06  300019
>>300016
Полезная нагрузка - корабль Аполлон. Сатурн-5 вытолкал его на траекторию к Луне (TLI - Trans-lunar Injection). Аполлон сам затормозил у Луны, затем:
- лунный модуль отстыковался от командного и еще раз затормозил, сойдя с орбиты
- приземлился на Луне
- взлетел (при этом он сам был двухступенчатым)
- сблизился и состыковался с командным на лунной орбите обратно
- всё это полетело к Земле и шлёпнулось в атмосферу прямо с размаху на 11км/с. (лунный модуль опять отстыковался перед входом в атмосферу - только командный мог выдержать реентри)
Аноним 15/06/16 Срд 17:20:16  300044
>>300019
>всё это полетело к Земле и шлёпнулось в атмосферу прямо с размаху на 11км/с
А сейчас, когда человеков с МКС на Землю возвращают, спускаемые аппараты с какой скоростью входят в атмосферу? Всё же 11 км/с - не хуй собачий: очень сильный удар об эту самую атмосферу должен получиться.
Аноним 15/06/16 Срд 17:46:29  300120
>>300016
Хах, вспомнил второй. Навеяно нейтронными звездами выше в треде. Собственно как они появляются? Суммарно нейтральный или около того ионный коктейль из легких ядер и электронов сжимается под действием гравитации после определенных этапов звездной эволюции и электроны впитываются в протоны, преодолевая электростатическое отталкивание или как оно там называется. Окей, но почему нейтроны не впитываются друг в друга? Что им мешает, если им даже никакого отталкивания преодолевать не приходится? Ну то есть я понимаю, что полноценное понимание состава нейтронной звезды - вопрос открытый. Вырожденный газ и все такое, но что мешает нейтронам собственно впитываться друг в друга? Сильное взаимодействие? Если да, то как? Квантовые эффекты? Если да, то какие? Или все это тоже вопросы открытые?
>>300019
Ну почти все угадал, только вот что удивляет, так это то, что назад снова летели на 11-ти? Нахуя или почему? Движки работали или земное притяжение сработало?
Аноним 15/06/16 Срд 18:26:01  300227
>>300120
>Что им мешает, если им даже никакого отталкивания преодолевать не приходится?
Сильное взаимодействие, одна из четырех фундаментальных сил. Со стороны это выглядит как давление вырожденного фермионного газа.
http://www.astronet.ru/db/msg/1188472

>Экспериментальные данные физики высоких энергий показывают, что с уменьшением расстояния между нуклонами ядерные силы притяжения сменяются силами отталкивания. Поэтому при плотностях <...> давление вещества оказывается больше, чем для газа невзаимодействующих нейтронов, способность вещества противодействовать сжимающей его силе тяжести увеличивается. В результате <...> повышается до указанных выше пределов, (1,4-2,7) <...>. Кроме того, отталкивание нуклонов с избытком компенсирует эффект, замедляющий рост давления с увеличением плотности, - рождение новых частиц (мезонов, гиперонов).
Аноним 15/06/16 Срд 18:48:35  300248
>>300227
То есть сильное взаимодействие при слишком большом сближении нейтронов меняет знак? Ох блять как все непросто. Ну ладно, спасибо.
Аноним 15/06/16 Срд 19:31:09  300263
>>299743
hope-x Хуево ты смотрел аниму
Аноним 15/06/16 Срд 19:55:22  300264
>>299548
Двачую вопрос
Аноним 15/06/16 Срд 20:08:59  300269
>>299940
Хуйню он написал из больной головы.
Аноним 15/06/16 Срд 20:11:55  300270
>>300005
В Роскосмосе модно сейчас ракеты освящать. Это единственное шаманство, которое используется.
Аноним 15/06/16 Срд 21:03:46  300281
>>300270
Это помогает? Есть статистика?
Аноним 15/06/16 Срд 21:06:33  300283
>>300281
Нет, не помогает.
Аноним 15/06/16 Срд 21:18:49  300287
>>300283
Зачем тогда освящают?
Аноним 15/06/16 Срд 21:22:29  300289
>>300287
Шоб потом сказать если что "ну мы тип все что могли сделали, даже освятили, а она все равно на старте пизданула"
Аноним 15/06/16 Срд 21:23:35  300292
(498Кб, 500x262)
>>300289
Аноним 15/06/16 Срд 21:41:43  300295
>>300264
На Naked похоже. Я сам не смотрел, но вроде такая показательная роль маргинала У Тьюлиса, часто нарезки с ним видел.
Аноним 15/06/16 Срд 21:58:44  300298
>>300014
То есть это "невозможно" упирается лишь в отсутствие адекватной теории? Утверждать, что что-то невозможно из-за отсутствия знаний об этом абсурдно, разве не так?
Аноним 15/06/16 Срд 22:21:49  300306
>>300298
Нет, не так. Абсурдно думать, что ты можешь чем-то управлять, не понимая, как это устроено и работает. То есть не имея на руках рабочей теории. Алсо, он нигде не говорил, что "невозможно".
Аноним 15/06/16 Срд 22:47:52  300312
>>300306
>искусственную гравитацию в разряде реализуемых на данный момент вариантов не рассматривает
А это что значит? Из данной фразы вытекает, что невозможно управлять гравитацией из-за отсутствия знаний о ней. Но в то же время ее глубокое понимание не гарантирует возможность контроля. В любом случае, при отсутствии истины, утверждать, что можно гравитацией управлять или нельзя очень странно.
Аноним 15/06/16 Срд 23:01:31  300316
>>300263
Точно похоже.
Чего хуево-то, там вроде не упоминалось про него, стояла себе можелька, ну и ладно
Аноним 16/06/16 Чтв 03:13:54  300346
А что если в черной дыре нет сингулярности ?
Вот представим нейтронную звезду. В ней нейтроны как мягкие мячики очень плотно лежат друг к другу. Потом 2 такие нейтронные звезды столкнулись и общая гравитация стала критической, шарики просто очень сильно сжались(каждый протон ведь состоит из 3-х кварков, между которыми явно есть какое то расстояние), и сфера стала меньше радиуса шварцшильда, допустим на 30%. И никакой бесконечной сингулярности с бесконечной плотностью не появляется, просто сверхплотная кварковая сфера, имеющая такую огромную массу что даже свет не может её покинуть
Аноним 16/06/16 Чтв 03:18:06  300347
>>300006
Я больше скажу, битард построил стройную схему, дал обоснование - вот разве что свое обьяснение недал. И все это посреди треда тупых вопросов, забитого платиной, сумасшедшими и школьниками. Спейсач торт.
Аноним 16/06/16 Чтв 03:19:38  300348
Хотя если подумать
Свет имеет постоянную скорость, и его ничто не может остановить, в т.ч. гравитация.
Просто в сверхмассивных объектах как черная дыра время идет очень медленно, и частота исходящих импульсов скорее всего не превышает нескольких Герц, именно поэтому она то и черная, а не потому что фотоны засасываются гравитацией
Аноним 16/06/16 Чтв 03:49:36  300351
>>300346
В ЧД материя падает из за ускорения больше критической. Т.е. какая бы сила не была она будет меньше или равна если свет с. За горизонтом же ускорение Больше или равно с.
>>300348
1 герц это 300 000 км радио волны. 2 герца 150 000 км.
Аноним 16/06/16 Чтв 03:50:25  300352
>>299130
Нахуй вам это красное каменистое говно сдалось? Лучше бы амс к Нептуну отправили или превозмогать к Венере.
Аноним 16/06/16 Чтв 03:50:38  300353
А может ли быть, что квазар это белая дыра?
Аноним 16/06/16 Чтв 03:56:04  300354
>>300352
Меркурий, нептун, уран - слишком ёба, хотя бы с финансовой т.з. Сатурн промежуточное туда редко летают.
Юпитер исследуют, только так.
Венеру и так и следуют, но спускаться влом и не готовы ещё.
Остаётся Марс и Юпитер. Но марс проще поэтому марс.
Аноним 16/06/16 Чтв 04:38:09  300356
>>300354
>слишком йоба
А к Плутонам и Харонам летать так им норм.
Аноним 16/06/16 Чтв 07:22:44  300361
>>300356
Нет, сложно. Но на Плутон не летали ещё, к тому же это пролётная миссия, а не орбитная, а это в разы проще. Для Уран, Нептун не требуется пролётная.
Аноним 16/06/16 Чтв 08:51:33  300364
>>300361
Но ведь у Уранов и Нептуняш орбиту получить же проще, чем у всяких Меркуриев?
Аноним 16/06/16 Чтв 09:27:29  300366
>>300353
Никто не знает, чем может быть белая дыра. Её существование - это теоретический вывод из решений Шварцшильда и Керра по ЧД. Есть мнение, что образование черной дыры в одной вселенной образует белую дыру в другой. Но, опять же, повторюсь, все это - лишь теоретические выкладки. Каких-либо практических способов обнаружить белую дыру, в отличие от ЧД, не существует.
Квазары же - по своей сути - сверхмассивные ЧД, которые, из-за гигантской аккреции излучают мощнее, чем все звезды отдельной галактики.
Аноним 16/06/16 Чтв 09:54:46  300369
Спейсаны, есть одна теория, которая сложилась у меня в голове, и не дает мне покоя уже несколько дней. Заключается она в том, что черные дыры не должны быть черными. Сейчас объясню.
По решению Шварцшильда, у ЧД есть гравитационный радиус, который зависит от ее массы, и равняется двойному произведению гравитационной постоянной на массу ЧД, деленному на квадрат скорости света.
На черную дыру постоянно падает излучение, в том числе, в спектре видимого света, от других звезд, галактик и прочих источников. Часть излучения проходит мимо, часть попадает в саму ЧД и оттуда выбраться не может. Но есть такая часть излучения (фотонов), которая может попасть на орбиту ЧД и вращаться там.
Далее. Хокинг доказал, что благодаря квантовому туннелированию ЧД должны излучать частицы. Следовательно – испаряться и терять свою массу. А если ЧД теряет свою массу, пусть, и ничтожно малую, её гравитационный радиус, по Шварцшильду, должен уменьшаться. А с уменьшением радиуса, те фотоны, которые были захвачены ЧД на её орбиту, должны с неё высвобождаться.
Таким образом, помимо излучения Хокинга, если оно действительно существует, ЧД должна излучать, в том числе, и в видимом спектре, что перестает делать ее черной.
Говном сразу не мажьте, сделайте скидку на то, что я гумманитарий. Как вам теория? Где я проебался?
Аноним 16/06/16 Чтв 10:25:26  300374
>>300369
Подозреваю, что скорость испарения чд слишком мала, что бы давать адекватное свечение. Таким образом большую часть своего существования чд просто черная сфера.
Аноним 16/06/16 Чтв 10:38:04  300378
>>300298
>что что-то невозможно из-за отсутствия знаний об этом абсурдно, разве не так?
Перечитал свой пост еще раз и, как и думал, ни разу не увидел слова "невозможно" или его производных. Не удастся и непреодолимый я использовал в контексте собственного взгляда на мир. Наука не оперирует категориями возможно/невозможно. Подобный догматизм был присущ религиозноориентированной науке средних веков. Ты сейчас волен следовать любой своей псевдонаучной похоти, и приверженцы официальной линии даже высмеивать тебя не будут, потому что слишком много уже таких фриков было. Да и Эйнштейна по началу за одного из фриков приняли, всякая хуйня случается. Научному сообществу присуща некая инертность и консервативность. Однако это уже издержки любой широкоорганизованной человеческой деятельности. Но в официальной науке есть принципы научного подхода. Наука не занимается тем, что "возможно", а занимается тем, что может пощупать здесь, сейчас или в ближайшей перспективе. Занимается тем, что может запустить на БАК, занимается тем, что может разглядеть Хаббл. Категориями возможно/невозможно стараются не оперировать. Относительно крепка еще память об историях вроде Джордано Бруно. Никто тебе в официальной обстановке не скажет, что копать специальным йоба экскаватором нельзя нейтронную звезду, а потом из полученного нейтрония строить спэйсшипы. Но тут есть две категории. Первая - невозможно на современном этапе развития технологий. Вторая - невозможно в рамках современной научной модели мироустройства. Так вот копать нз йоба экскаватаром - первая категория. Копать еще более йобистым экскаватором чд - уже вторая. Так вот искусственные поля гравитационного тяготения без соответствующих масс - именно вторая категория.
Аноним 16/06/16 Чтв 10:48:11  300380
>>300378
Опять какая-то софистика. Спасибо, что хоть в конце сказал по делу.
Аноним 16/06/16 Чтв 10:52:23  300381
>>300351
Воу воу, паринь. Ты сейчас в одном посте и силу и ускорение в метрах в секунду выразил. Негоже такой ересью на этой доске срать. Горизонт событий означает, что скорость убегания под ним превышает с. Ускорение в таком случае становится сугубо вторичным параметром.
Аноним 16/06/16 Чтв 13:00:08  300388
Почему бы не поставить на ракеты системы аварийного спасения полезной нагрузки (спутники, оборудование и т.д.)? Тонны же металла, труда и времени теряются в случае аварии. Или накрайняк, поставить туда черные ящики для подробного расследования причин аварии.
Аноним 16/06/16 Чтв 13:10:19  300389
>>300388
Дорого, и весит много. Проще застраховать, если везем просто кусок железа. А черные ящики там и так есть.
Аноним 16/06/16 Чтв 15:17:33  300405
>>300369
Свет имеет постоянную скорость, и его ничто не может остановить, в т.ч. гравитация.
Просто в сверхмассивных объектах как черная дыра время идет очень медленно, и частота исходящих импульсов настолько низка, что света просто мы не видим(свет же тоже имеет свою определенную частоту в гигагерцах), именно поэтому она то и черная, а не потому что фотоны засасываются гравитацией
Аноним 16/06/16 Чтв 15:21:32  300406
>>300405
Ебанько, ЧД изучают на всех диапазонах и на всех длинах волн. И они ни в каких ничего не излучают.
Аноним 16/06/16 Чтв 15:22:59  300408
>>300406
пруф давай, шелестеть так все подряд можно
Аноним 16/06/16 Чтв 16:30:05  300416
Щас вы охренеете но я пришел к очень немаловажному выводу.
Вторая космическая скорость - для земли 11 Км/с.
Для черной дыры - от 300 000 Км/с (как любят писать в статьях)
Но что есть эта вторая космическая скорость ? Это по сути просто скорость единовременного пинка под жопу предмета чтоб вылететь с планеты.
Мы можем не набирать 11 км/с, чтоб свалить с земли, мы можем на обычном тихоходном реактивном ранце с неё улететь при скорости даже 100м/с, просто времени уйдет чуть больше. Ведь притяжение у нас всего 10м/с, но никак не 11км/с

А что черная дыра ? Да тоже самое, с неё не могут улететь бедные каменюги, потому что у них нет реактивного двигла чтоб набрать хоть какую то скрость. То есть камни могут получить только единовременное ускорение, но оно никогда не привысит второй космической скорости на черной дыре это понятно. Но если у нас будет космический корабль с движком, то он вполне сможет улететь из под горизонта событий. Даже если там гравитация в 100.000 раз больше чем на земле (реально меньше), то посчитаем что это примерно 10м/с(g на земле)*100.000=1000 000м/с=1000км/с это ускорение свободного падения на черной дыре. Обычному кораблю будущего превысить это скорость будет не так уж проблематично. Только он вылетит в далеком будущем через несколько миллиардов лет после влета из-за разницы во времени.
Аноним 16/06/16 Чтв 16:50:13  300418
Читая всякое по астрономии, почувствовал, что мне нужно обновить свои знания физики. Электрические/магнитные поля, плазма, оптика. Чего бы почитать?
Аноним 16/06/16 Чтв 17:52:09  300421
>>300416
Вспомни картинку с видом от 5 метров от ЧД и скажи где за ГС верх.
>>300364
На какой высоте по твоему Уран, Нептун. Даже вояджеры десятилетие летел, а что бы орбиту это надо или ещё десяток лет или пилить ёбу среди всех ёб, недоступную сегодня.
Аноним 16/06/16 Чтв 17:57:39  300422
>>300421
На горизонте событий вся окружающая вселенная визуально сжимается в маленький блестящий диск. Туда и пиздовать.
Аноним 16/06/16 Чтв 18:00:43  300423
>>300405
Свет - это электромагнитное излучение определенной частоты. Поэтому если свет другой частоты, то это уже низкая не свет.
Блядь, перечитал и сам с себя проиграл. Мама, чому я объясняю как хач
Аноним 16/06/16 Чтв 18:01:14  300424
>>300423
*нихуя
фикс
Аноним 16/06/16 Чтв 18:04:54  300425
>>300406
> ничего не излучают
Старобинский, Зельдович и Хокинг говорят, что ты хуй
Аноним 16/06/16 Чтв 18:17:16  300426
>>300408
Тебе пруф нужен на существование радиотелескопов, милая?
Аноним 16/06/16 Чтв 18:20:38  300427
>>300425
Как только зарегистрируют, сразу приходи. А пока сиди молча. Алсо, если ты найдёшь в себе силы разобраться в этом излучении, то с удивлением для себя узнаешь, что оно исходит не из под горизонта.
Аноним 16/06/16 Чтв 18:32:14  300430
>>300416
Все правильно.
Аноним 16/06/16 Чтв 20:42:37  300436
>>300426
И понятно что они ничерта увидеть немогут, ибо время в черной дыре практически не идет и любое излучение из нее вырывается с частотой меньше 10 гц, а такие волны до нас физически дойти не могут
Аноним 16/06/16 Чтв 20:58:14  300437
Опять Чернодыр принимает пополнение? Давайте перекат уже.
Аноним 16/06/16 Чтв 21:38:00  300443
>>300436
Ты свои фантазии прекращай, заебал хуйню молоть.
Откуда ты взял именно 10 Гц? Как ты их высчитал?
>а такие волны до нас физически дойти не могут
Какие физические законы этому препятствуют? Почему 30 Гц доходят, а 10 не доходят? Что с ними случается?

Аноним 16/06/16 Чтв 22:46:51  300451
>>300443
Расколбашиваются об кефир.
ПЕРЕКОТ Аноним 16/06/16 Чтв 23:48:51  300456
Перекатываемся.

>>300455 (OP)
https://2ch.hk/spc/res/300455.html
Аноним 18/06/16 Суб 02:05:30  300728
>>300295
>Naked
пасиб, дорогой
Аноним 26/12/16 Пнд 22:57:34  319974
>>299864
Люто, бешено космочую вопрос.

[Назад][Обновить тред][Вверх][Каталог] [Реквест разбана] [Подписаться на тред] [ ] 586 | 51 | 167
Назад Вверх Каталог Обновить

Топ тредов
Избранное